"What Evidence is There Against the Existence of God?"

Dr. William Lane Craig asks this question in his debates. Let me attempt to answer it.

In the first place, what is the evidence against the existence of fairies or unicorns? If by looking and not seeing any isn’t considered evidence against their existence, then I don’t know what is required here. Let Dr. Craig first provide evidence against the existence of fairies or unicorns and I’ll provide evidence against the existence of God. Someone cannot provide evidence against the existence of an non-entity, since if it doesn't exist then it cannot leave any traces of its non-existence for us to examine. Think about this.

Now I do happen to think there is evidence against the existence of the Christian God, since that God depends upon the revelation found within the pages of the canonized writings in the Bible. There is the empirical evidence of intense undeserved suffering in the world which cannot be explained by a perfectly good omnipotent creator; there is archaeological evidence against the Biblical stories of the world-wide flood, the Exodus and the conquest stories in the Bible; there is geological evidence showing the earth has existed for 5 billion years; there is biological evidence showing one species evolved into the next one which disconfirms there was ever a time when there was no death in the Garden of Eden; there is psychological evidence that no wrathful God could exist given the fact that we believe and behave as we do based upon early childhood experiences; there is neurological evidence in that strokes and seizes disconfirm the notion of a soul; there is historical evidence against the believability of the virgin birth story, Satan, hell and the bodily resurrection of Jesus from the dead too. Christians will try to dispute this evidence and/or try to show it doesn't amount to much. I vehemently disagree, but it is evidence, plenty of it. And there is more I haven't mentioned. The evidence is against the God we find in the Bible, period.

197 comments:

Charlie said...

Evidence against the existence of unicorns and fairies: 1) we'd expect to see such entities were they to exist, but we don't, so they probably don't exist; and 2) most people who write about these entities would already admit that they're fictional (yourself included), demonstrating that there is no serious ontological commitment in the first place. The same cannot be said about theism or agnotheism.

I do think the evidence you mentioned all ads up as a compelling reason to reject the literalist biblical conception of a deity -- and as you know that is not to be equated with theism per se.

Anonymous said...

Fine Charlie. I read the link. Would you please tell me why Dr. Craig asks atheists in his debates for evidence against the existence of God if he might turn right around and say there can be no evidence against a spiritual being like the God of theism? If by definition the God of theism is a spiritual entity then Dr. Craig needs to decide beforehand whether or not such a God admits of physical evidence. If not, then he shouldn't be laying down such a challenge. If so, then he needs to specify what kind of evidence he thinks is different than the kind I speak about (i.e., looking and not finding), which would be the same thing he would do to show there are no fairies, unicorns, or flying teapots.

Anonymous said...

And Charlie, I think Theodore Drange's argument that the God we find in the Bible is the kind of God we should expect to find, since he desires this of us. We should expect to find him if we search for him. So where is he?

Charlie said...

John,

he might turn right around and say there can be no evidence against a spiritual being like the God of theism?

He doesn't do that. As far as I can tell, Craig thinks that theism can in principle be falsified through (1) logical inconsistencies or (2) probabilistic conclusions based upon natural, observable phenomena. And in fact, this is what naturalists have been trying to do for centuries. It's just that many (Craig included) think such naturalists have been generally unsuccessful.

Anonymous said...

Charlie your #1 is irrelevant to his question. He's asking for evidence. I responded directly to his question. And I also provided a short list of evidence for your #2.

Charlie said...

And Charlie, I think Theodore Drange's argument that the God we find in the Bible is the kind of God we should expect to find, since he desires this of us. We should expect to find him if we search for him. So where is he?

I would agree that the literalistic biblical deity probably doesn't exist. The rational thing to do would be to expand the scope of one's search within the bounds of natural reason, rather than committing to a full-blooded atheism and rejecting theism simpliciter.

Adrian said...

What about something like Dawkin's Ultimate 747 which would apply to any god?

Related to that, if a god is omnipotent it must be finely ordered and complex making it vastly more improbable than the Universe coming into existence fully formed. It demands an explanation.

If the god is complex in any sense (capable of thought or knowledge, for instance) it means the god must be composed of smaller elements meaning it isn't eternal and could only have evolved making it, at most, a powerful alien but not a god.

Perhaps someone has done a better job of explaining these in a simple, pithy form but these have struck me as important arguments against a god.

Charlie said...

Charlie your #1 is irrelevant to his question. He's asking for evidence. I responded directly to his question. And I also provided a short list of evidence for your #2.


No, (1) is not irrelevant. Logical evidence counts as evidence just as much as observable evidence does. If you think 'evidence' means only things available to the senses, then you're defining that term in a way that most mathematicians, physicists, biologists, cosmologists, etc., etc., would reject.

As for (2), I already granted that your evidence is compelling with respect to rejecting a particular, narrowly conceived biblical deity -- but again, as you know, this has little or no bearing on the general question of whether a God exists.

kiwi said...

Tyro,

God is simple, because Aquinas said so.

;)

Anonymous said...

Charlie, define evidence as you will then. I wonder what Craig means by it? Besides, I can think of very few, if any, logical disproofs of things, so it's still irrelevant since entities cannot be logical proved not to exist by virtue of "logical inconsistencies," as you said (square circles, yes; unicorns, and God, no).

It's okay if you are, but you sound like a deist. A Christian theist does not follow the light of natural reason for his or her beliefs. A Christian theist is someone who stands within the revelational community of believers in the tradition stemming from the Biblical texts.

Charlie said...

If the god is complex in any sense (capable of thought or knowledge, for instance) it means the god must be composed of smaller elements

Supporting argument for this assumption?

The complexity of some existent does not necessarily imply smaller components of that existent, nor does it imply any composition relation for that matter. Individual substances with no parts (save for one improper part) can be complex in different ways, yet they would lack smaller components. I think Dawkins's argument here, at least as you've construed it, crucially hinges upon what he means by terms like 'complex' and 'part' as well as the scope of his quantification. It is by no means clear.

Charlie said...

John,

Charlie, define evidence as you will then.

My definition is not limited to observable entities; and most scientists nowadays agree with this. Craig would also agree with it since he things there can be (unobservable) logical evidence against things. You can tell by reading some of his debates and articles.

Besides, I can think of very few, if any, logical disproofs of things, so it's still irrelevant since entities cannot be logical proved not to exist by virtue of "logical inconsistencies," as you said (square circles, yes; unicorns, and God, no).

I am having trouble making sense of this sentence, so I won't address it.

I think it's true that most Christians and most atheists and most humans in general, regardless of their ideology, have difficulty following the natural light of reason. Nothing new.

Adrian said...

Tyro:
If the god is complex in any sense (capable of thought or knowledge, for instance) it means the god must be composed of smaller elements

Charlie:
Supporting argument for this assumption?



In essence, the ability to store a single bit of knowledge, the answer to a single yes/no question, means that an entity must have the ability of storing data in one of two potential states. The data-storage "system" is the collection of all of these elements which, because it's an aggregate of different state components is complex by any definition I'm aware of. It is composed of the individual states.

This doesn't even touch upon the mechanism for altering, retrieving and interpreting these states!


There's an outline. Was there something more specific you were thinking of?

kiwi said...

Tyro,

You should read Plantinga's review of "The God Delusion".

I don't find it convincing, but it does answer your point. Simply put, Plantinga claims that God is a "Spirit" and not like a material object like a computer, so God's knowledge is not "stored" anywhere.

Adrian said...

Kiwi,

Yeah, I read that. I generally try to avoid the whole thing because it just provokes rants. (Fair warning.)

Frankly I don't see how it begins to answer these objections. He tries to obscure the questions and tosses out some special pleading but I don't think he does any any damage. At times his argument seems to lack any intellectual honesty and looks to grasp at any straw that comes along.

For instance, to deal with the argument that a god must be complex he says "According to much classical theology (Thomas Aquinas, for example) God is simple, and simple in a very strong sense". Well whoopdy-flipping-do. I don't give a frog's fart what Aquinas says, I only care if he's got some reason and we don't see any reasons, merely assertions.

P says "But of course God is a spirit, not a material object at all, and hence has no parts" which again doesn't begin to touch upon the issue. Give me a reason to think that something which has no parts is capable of knowledge, thought or action. Without this, the best you can do is say that God is simple and therefore has less intelligence and information content than a rock (less than even a proton) and less capability for influencing the world than a single electron.

No, what's necessary is some argument that says that you can be both "simple" and still have the attributes of a god and Plantinga is either oblivious to this or, more likely, uninterested in dealing directly with these questions hence the multi-page smoke screen which serves only to confuse and obfuscate but which was never intended to enlighten.


(I'm not just cherry-picking, he does this throughout. Plantinga's response to whether God is improbable? Why to just state that God isn't improbable: "So why think God must be improbable? According to classical theism, God is a necessary being." Does he bother to argue that a "necessary being" could have any of the attributes ascribed to him? No, of course not. Does he for once consider how probable or improbable god is? No, don't be silly. Assertions, quotes from antiquity and the smug implication that Dawkins hasn't bothered to learn his theology works well enough for him so why actually deal with the arguments? Maybe science and its genuine pursuit of truth and high ethical standards has spoiled me but this sort of behaviour really revolts me. If anyone has a genuine response I'm interested but this is nonsense.)

Adrian said...

kiwi,

A less rant-y answer:

God's knowledge is not "stored" anywhere.

Not good enough. When confronted with a strong positive argument, this is just special pleading. Present an argument that one can have knowledge without storing it else concede.

David Parker said...

Charlie, I was about to post a link to that Maverick Philosopher article on Russell's teapot and then I noticed you already did. :-)

kiwi said...

Tyro,

I totally agree with what you've said. I do think theists have a lot of explaining to do if they want to claim that God is simple. Just to clarify, I wanted to point out that Plantinga discusses the issue, not that what offers is convincing.

Anyway, don't be afraid to rant, your rant was very interesting. ;)

mathyoo said...

a better argument than fairies or unicorns would be asking Dr. Craig to prove that Zeus, Allah, Krishna, Odin, Xenu, etc. do not exist. All of those either have or have had believers just as ardent as any Christian, yet there is exactly as much evidence for the existence of any of them as there is for the Christian God. That is, to say, no evidence at all. Yet Dr. Craig, as well as pretty much every other Christian on the planet, would deny the existence of those other Gods but not their own.

Then there's the issue of the definition of the Christian God. In order to be able establish the existence of any given entity, you have to be able to define that entity somehow. If the definition is logically incoherent, as most Christian definitions of their God seem to be, it's not any more difficult to disprove God than it is a square circle.

Charlie, per your first comment, we'd also expect to see a God if it existed. Yet, we don't. We only have a few very questionable, unverifiable accounts by people claiming to have spoken to God.

David Parker said...

Charlie, per your first comment, we'd also expect to see a God if it existed.

You'd expect to see an infinite being?

mg01 said...

The burden of proof lies with the one making the claim. Not the other way around. It is not possible to prove a negative.

The question then is what evidence is there for the existence of God.

For God in general, cosmological, ontological, and tautological arguments all fail.

For the personal, prayer answering God of Judeo-Christian tradition, there is the protestant cited evidence is the Bible, which however, is not evidence, but a collection of claims that are either anecdotal personal revelations or post-hoc reasoning based on cultural bias and circular logic.

If anyone today claimed he had spoken directly to God and was given instructions to do X, would anyone take him serious. (Actually some probably would, but that makes the point doesn't it.) Why then would we accept the idea that anyone from Adam to Zechariah in the Bible did either, while at the same time rejecting the claim of Muhammad or figures of any other culture.

As an ancient sacred text the Bible tells us that long ago in a land far away, magical things happened. There is a word for this genre of literature.

For a phenomenon to be detectable, it has to have an effect within the reality of nature we experience. But before any such effect can be attributed to supernatural causes, all natural ones must first be ruled out. Even with the 0.1% of events that can't be ruled out by natural explanations, what practice benefit is an effect that manifests itself so infrequently.

Charlie said...

The burden of proof lies with the one making the claim. Not the other way around. It is not possible to prove a negative.

The question then is what evidence is there for the existence of God.


infidels.org from 1999 called; they want their specious reasoning back.

David B. Ellis said...


Evidence against the existence of unicorns and fairies: 1) we'd expect to see such entities were they to exist, but we don't, so they probably don't exist; and 2) most people who write about these entities would already admit that they're fictional (yourself included),


Belief in fairies is uncommon in the US.

In Iceland, however, I've read that over 50% of the population belief in "hidden folk" like fairies, elves, trolls, etc.

We wouldn't necessarily expect to see evidence of fairies if they choose to stay hidden---exactly the response christians give for why God and angels don't make their existence known in any empirically observable way.

David B. Ellis said...


The rational thing to do would be to expand the scope of one's search within the bounds of natural reason, rather than committing to a full-blooded atheism and rejecting theism simpliciter.


I'm perfectly willing to entertain the idea that a god of some variety exists---if rational grounds for thinking so are provided.

I'd do the same for werewolves, vampires, fairies and all the rest of the things I currently see not an iota of reason to believe in.

Until good reason to believe is provided, though, I see no reason to give such things any credence at all---I don't concern myself about whether a god or an afterlife exists in exactly the same way I don't bother carrying garlic with me at night just in case vampires turned out to be real.

Charlie said...

Tyro and Kiwi,

Dawkins needs the following claim for his argument (as construed here) to go through: necessarily, if a thing is complex in any way, then that thing has smaller parts composing it.

Now, all that needs to be true for Dawkins's argument to fail is the proposition that, possibly, there's at least one thing that is complex in some way and that thing isn't composed of smaller parts. All mereologists believe that this is possible.

(A thought experiment readily demonstrates why. Imagine a world in which a string ('string' here is just a placeholder for any simple object you want to think of) located at some region of space at t1, and then located at another region* of space at t2. The string is complex in at least one way: it bears different spatial and temporal properties. Yet it lacks smaller proper parts. Therefore, it's possible for there to be a simple object that is complex in some way or another.)

Many think that souls are simple in that they lack physical proper parts, but complex in that they are the subjects of widely diverging experiences. Of course you'd dispute the existence of souls; but the point is, so long as it's possible for a simple object to be non-mereologically complex, then Dawkins's argument fails.

Charlie said...
This comment has been removed by the author.
Charlie said...

David,

I could be tiresome and say that if we looked hard enough, we should expect to find a "fairy" since they're purported material objects (if they choose to hide, we could expect to find them with infrared).

But I already know how this game goes. The problem for you is that the more qualifications you add to the beings you're imagining, the closer you arrive at genuine unfalsifiability. And if that's your goal, then there is still no plausible analogy to theism, which in its more philosophically refined forms is a falsifiable theory, the denial of which is capable of being supported by different sorts of evidence.

Jeff C said...

I do think the evidence you mentioned all ads up as a compelling reason to reject the literalist biblical conception of a deity -- and as you know that is not to be equated with theism per se.

Okay, if John's reasoning adds up to compelling evidence against a literalist biblical conception of a deity, then what what are you saying is left? Craig's question involves the Christian God. The Christian God is described in the Bible. Without the Bible there is nothing left of Christianity -- no Jesus, no Holy Spirit, no stories of Moses, Isaiah, the works. What would be left is some deity living in the minds of worshipers that is definitely *not* the Christian God. Do you agree? To re-iterate what John said, that sounds very deist in nature, definitely not Christian.

Charlie said...

David,

"I'm perfectly willing to entertain the idea that a god of some variety exists---if rational grounds for thinking so are provided."

Whatever their respective truth-values turn out to be, any serious participant in these debates would acknowledge that theism and atheism are both rational points of view, held by many brilliant philosophers and scientists on each side of the division.

People who go around claiming that theism can't be rational are just victims of Dawkma.

Charlie said...

Jeffery,

It's possible to believe in a God that isn't defined narrowly by a holy book. If our only choices were between a literalist notion of a biblical god and atheism, this would be a rather depressing place.

Jeff C said...

My definition is not limited to observable entities; and most scientists nowadays agree with this.

"Most scientists" isn't a particularly strong argument. And in fact, as somebody trained in the sciences, I disagree with that statement. Can you think of places where science uses un-observable evidence? I'm not sure there is such a thing. (For example, while science might not actually observe evolution over millions of years, they base their study of evolution on observations made today.)

Jeff C said...

"It's possible to believe in a God that isn't defined narrowly by a holy book."

Hi!

But then you're talking about "a" God, not necessarily "the" Christian God (which Craig believes in). Do you see the point I'm trying to make?

Jeff

kiwi said...

Charlie,

I'm not sure if Dawkins' argument (as presented in his book) is successful, however like Tyro I'd like to know how can a being be omnipotent and simple at the same time. I think to simply say that God is a spirit like Plantinga does is not a satisfactory answer.

Theists are quick to point out that the best explanation for X, Y, Z, etc is a god, but surely for an explanation to be satisfactory, we have to understand how he "works", at least partially. I feel god as an explanation is a non-explanation, if what is meant by "god" is basically a mystery.

Jeff C said...

A thought experiment readily demonstrates why. Imagine a world in which a string ('string' here is just a placeholder for any simple object you want to think of) located at some region of space at t1, and then located at another region* of space at t2. The string is complex in at least one way: it bears different spatial and temporal properties. Yet it lacks smaller proper parts. Therefore, it's possible for there to be a simple object that is complex in some way or another.

Charlie, I don't agree with your conclusion. This thought experiment doesn't show that it's possible for there to be such an object; it only shows that it's possible to imagine or conceive of such an object.

Einstein was famous for thought experiments. He liked to use thought experiments to help people understand things, but he didn't use these in his formal papers; for that he needed to provide a clear mathematical framework and then experimental evidence. So a simple thought experiment only goes so far.

Jeff

Charlie said...

Jeffrey,

Was that a sarcastic post?

If not, Einstein's theories of relativity were based to a significant degree on thought experiment and inference. (That's just a popular one.)

If you think every entity scientists believe they have evidence for is observable, then I'm suspicious of your claim to be a scientist.

Charlie said...

Jeffrey,

Conceivability is a fallible but firm guide to possibility. It's possible for the chair you're sitting on right now to have been located two inches to the left of where it actually rests. We don't need mathematical formulas to show this; conceivability is sufficient.

Jeff C said...

I'd like to know how can a being be omnipotent and simple at the same time.

When I used to be a Christian, I would tackle ideas like that by just punting and saying that God is too great and powerful for me to understand. That would basically give me a license to cheat on logical inconsistencies (although back then I didn't view it as cheating). A favorite of mine was that "logic exists in this universe; God lives outside the universe and can, therefore, defy logic."

The problem, of course, was that from there it would turn into a free-for-all where reason is totally left behind. Of course, I suppose I'm also proof of where that kind of thinking took me: I started to doubt the literal truth of the Bible, which slowly made the whole thing collapse. And today I don't buy into it at all and am no longer a Christian!

Adrian said...

Charlie,

I don't know anything about mereology so if that's a real objection can you provide more detail?

"Many think that souls are simple in that they lack physical proper parts, but complex in that they are the subjects of widely diverging experiences."

I'll give the same response I gave to Plantinga - demonstrate that souls exist and are both complex and simple otherwise this is just another example of theological phenomena that get crushed by the complexity argument.

You do have the start of an interesting counter example though:

The string is complex in at least one way: it bears different spatial and temporal properties. Yet it lacks smaller proper parts. Therefore, it's possible for there to be a simple object that is complex in some way or another.

Okay so first the location of an object in spacetime is not a property of the object so much as it is a property of spacetime. How would we go about conveying & storing information in such an environment? We could say "if there is an object at coordinates (x1,x2,x3,x4) then P(x) is true, else false" which just conveys a single bit. I think what you're getting at is this: "if object P's X-coordinate is x1 then true, else false." This leaves us with several problems. If we store more than one objects in space, we need to uniquely identify them which means that each object must have at least as many individual properties (separate from spacial location) as there are objects in space. If we are using simple objects, a 4-dimensional space is only capable of storing 4 bits of information. To store a billion bits of information, we would need a billion dimensions! (I'm simplifying things a lot, but you get the drift.)

The other, bigger problem comes with the question of data representation & retrieval. I think I've shown that to get by with objects that are simple we've pushed the complexity into the space it's stored in. We still need some means of representing this data, retrieving it and acting upon it. Let's say you discover a universe with 10*5000 dimensions with a single object in it. This universe could potentially represent 5,000 bits of useful information. How are you to know? How are you to retrieve the information without disturbing the state within this universe? The storage of meta-information and the ability to retrieve and interpret new information requires in itself a complex and highly ordered system like a Turing machine.

Let me guess, you think this can be done by using uni-dimensional, simple objects as well? Well it can't be turtles all the way down, this complexity (and worse, this high degree of order) is unavoidable and totally ignored by any apologist.


If that's a little abstruse, how about another example. What's the biggest number you can count to using your fingers? 10? Think again.

If our fingers are binary up/down and we also pay attention to which fingers are up and down, we can count to 2^10-1 or 1023. This is still a 2-dimensional counting with but let's add in a 3rd dimension. If we also consider whether we hold our hands up palm out or palm in that number grows to 2^12-1. We can increase this number higher and higher by checking whether our hands are in front of us or at our sides, by how many degrees from our body they are, and so on. We can check the angle of our fingers and on and on. Depending on the precision of our measurements, the number could grow astronomical.

However all of this requires some system of interpretation. If I stick one arm straight out, palm up with the first finger up, second one up half way and the fifth up a third, what number am I representing? We would have to have some means of conveying this information ahead of time, storing it (or remembering!), retrieving it on demand and comparing it to this situation. Even the simple act of holding up one hand with some fingers held up requires understanding the convention that we count 1 for each extended finger and 0 for all others. If you try to argue that we can store a large amount of information by some complex, convoluted spacial arrangement (which is what we're doing with our fingers), you just demonstrate how much extra complexity is required elsewhere. If we're measuring 10 different things when we look at someone's fingers, we still need some working memory to count, some means of writing & reading this working memory and some means of interpreting. All of this requires order and complexity.

You simply cannot get all of this with a simple, single entity. It is impossible.


But nice example. Much better than Plantinga.

Charlie said...

Jeffrey,

But then you're talking about "a" God, not necessarily "the" Christian God (which Craig believes in). Do you see the point I'm trying to make?

Right, I'm not necessarily talking about a God defined narrowly by any holy book.

Craig has a specific argument for the God in whom he believes -- which has to do with a resurrection. If he didn't have that argument, he could still present a case for theism.

Jeff C said...

Was that a sarcastic post?

No.

If not, Einstein's theories of relativity were based to a significant degree on thought experiment and inference. (That's just a popular one.)

Einstein's theories started out as thought experiments. But two things had to take place before they were widely accepted by the scientific community: He had to come up with a formal mathematical model (which include the development of an entire branch of mathematics to accommodate it) and (2) theoretical evidence. And we have plenty of theoretical evidence today of the truth of relativity. Had those two things not taken place, Einstein's work would have been left behind as just that: a thought experiment with little or no acceptance today.

If you think every entity scientists believe they have evidence for is observable, then I'm suspicious of your claim to be a scientist.

First, I'm not claiming to be a scientist; I'm claiming to have been trained in science, having completed graduate work in mathematics and physics.

Second, this might just be a miscommunication. I was responding to this:

"Charlie, define evidence as you will then."

to which you said:

"My definition is not limited to observable entities;"

I thought you were talking *evidence* being observable. I agree there are plenty of *entities* we can't observe that we have evidence for.

Larry Hamelin said...

You can just as easily use evidentiary reasoning to prove nonexistence as existence. It's just a matter moving the logical operators around and adjusting definitions.

If seeing this comment is evidence that the comment does exist, then not seeing a message labeled as posted by Albert Einstein is just as good evidence that no such comment exists.

The problem is as Loftus notes in the 2nd comment: God is not defined (at least by half-smart philosophers) in such a way that evidence can either prove or disprove the existence of God. Any logically possible observation is compatible with both the existence and non-existence of God.

With such a definition, the lack of evidence against God is no more epistemically relevant than the lack of evidence for God. It's actually a misnomer to say that such an unfalsifiable God does or does not exist: The label "God" with such a definition is simply irrelevant and meaningless.

Charlie said...

Tyro,

If you don't know anything about mereology (theory of parts and wholes), you might not be so quick to assert that an "object which is complex (in any way) must have smaller parts".

I've shown that to undercut Dawkins's argument one just needs the possibility of a non-mereologically complex object that lacks any parts (save for a single improper part: itself). I gave a thought experiment in favor of this possibility. Even if you don't believe properties exist, nothing hangs on it. As long as the object in my counterexample is complex "in any way" then it's sufficient to undercut Dawkins's premise; this is fairly easy to do without properties, but I'll leave it to the reader to construct her own.

If you can sum up the rest of your post in a more concise manner, I'll read the remainder. (Also, I have no idea what you mean by "if our fingers are binary up/down ...")

Charlie said...

Barefoot bum,

If seeing this comment is evidence that the comment does exist, then not seeing a message labeled as posted by Albert Einstein is just as good evidence that no such comment exists.

Not so, for the simple reason that it's possible for me to not notice an existing comment by Albert Einstein.

Absence of evidence is not evidence of absence. (Kind of a "duh" issue here.)

Charlie said...

Jeffrey,

"When I used to be a Christian, I would tackle ideas like that by just punting and saying that God is too great and powerful for me to understand. That would basically give me a license to cheat on logical inconsistencies (although back then I didn't view it as cheating). A favorite of mine was that "logic exists in this universe; God lives outside the universe and can, therefore, defy logic."

The problem, of course, was that from there it would turn into a free-for-all where reason is totally left behind."


Given such superficial tendencies in your thinking, I would not be surprised if you later came to embrace atheism.

Jeff C said...

If you can sum up the rest of your post in a more concise manner, I'll read the remainder. (Also, I have no idea what you mean by "if our fingers are binary up/down ...")

Tyro just means that you can find ways to count on your fingers to very high numbers. One way is to use what's called binary numbers, where each finger can be either up or down, representing "1" or "0". There are 1024 possible combinations of having your fingers up or down, which means you could use this way to count on your fingers from 0 to 1023.

Jeff C said...

Given such superficial tendencies in your thinking, I would not be surprised if you later came to embrace atheism.

I don't even know you but you seem like a terribly mean person. Superficial tendencies???

And when did I say I'm an atheist?

Charlie said...

Jeffrey,

I did not intend to be rude there. You yourself seem to disapprove of the way you thought as a Christian. It certainly looks superficial to "leave reason behind" and say that God is beyond logic. If that's not intellectual superficiality, I don't know what is.

And I never said you were an atheist. I said I wouldn't be surprised if you were, given such tendencies.

Charlie said...

Tyro just means that you can find ways to count on your fingers to very high numbers. One way is to use what's called binary numbers, where each finger can be either up or down, representing "1" or "0". There are 1024 possible combinations of having your fingers up or down, which means you could use this way to count on your fingers from 0 to 1023.

Got it, thanks. And how is this relevant to the counterexample I outlined?

Jeff C said...

It certainly looks superficial to "leave reason behind" and say that God is beyond logic...

Oh, sorry, I thought you were saying my thinking as a whole is superficial; I just re-read what you typed and realize you were referring specifically to my thoughts while I was a Christian.

So yes, it was rather superficial, but it was a way to try and overcome difficulties that I saw in the logic of what they were teaching me at church, a way to make it "work" for me. They would teach me something at church that seemed completely impossible, so I would twist it in my mind into something that seemed possible, to make it work. Because I knew that God and Jesus *must* be real, so the only way to go was to make the science or logic fit. And sometimes that meant abandoning logic and science altogether in my mind.

In fact, I'm convinced that a lot of Christian apologists do exactly that; they see something that seems unbelievable and then try to bend it into something that works. Probably the most extreme example are the young earth Creationists like those at the Creation Museum who see something scientific (such as fossils of dinosaurs and carbon dating) and figure it *must* fit with the Bible because in their eyes the Bible simply can't be wrong. So they twist the science to force it to fit. I was quickly headed that way until it all finally just broke and I had no choice but to let go of one thing or the other -- let go of Christianity, or let go of science. I let go of Christianity. :-)

Jason Long said...

Well, it's probably too late for me to chime in here, but I think it's always good for us to take the first step of seeing if unicorns or Zeus will fit into a question such as this so that it can be turned around on the one asking. And I believe Loftus is right: there is no more evidence against unicorns than there is against God. Wide belief is not evidence. No continued presence is no more a compelling piece of evidence against unicorns than it is against God.

Yes, the only evidence we are left with is the Bible. Without the Bible (or parts), no one would believe in the Judeo-Christian God. And there is compelling evidence in every category to consider it a fraud. But I'm not sure how strokes and seizures would disconfirm a soul.

Overall, an excellent post.

Adrian said...

Charlie,

If you don't know anything about mereology (theory of parts and wholes), you might not be so quick to assert that an "object which is complex (in any way) must have smaller parts".

If it's such a well-developed discipline, it shouldn't be hard to offer counter-examples or counter-arguments.

I've shown that to undercut Dawkins's argument one just needs the possibility of a non-mereologically complex object that lacks any parts (save for a single improper part: itself). I gave a thought experiment in favor of this possibility.

And I showed why your thought experiment is invalid. Would you care to defend it?

As long as the object in my counterexample is complex "in any way" then it's sufficient to undercut Dawkins's premise; this is fairly easy to do without properties, but I'll leave it to the reader to construct her own.

No, you're wrong. In your example your simple object was not complex and it was not capable of storing any data. The only way you were able to make it appear as if it could was by moving it in a complex space or by saying the existence/non-existence of this object is information. Again, the only information stored is in the space, not in the object. As I showed, this means that you need an an extra dimension for each bit of information to be stored and the contents inside to be precisely ordered. Plus I showed how you need a very complex, ordered system to make sense of this space.

So your example was not a counter-example, it just served to illustrated why you need a complex, ordered, composite in order to store and retrieve information.

If you can sum up the rest of your post in a more concise manner, I'll read the remainder.

It was your example, I assume you wanted to defend it. I don't know what you want me to summarize, but I can give bullet points.

1. Your object was simple but it stored no information or state
2. All state information was a property of the space you chose
3. If you use simple entities, your space becomes extremely complex with a new dimension needed for each bit of information
4. Given any representation of information, we need a complex, ordered composite to retrieve and operate on this information

(Also, I have no idea what you mean by "if our fingers are binary up/down ...")

I mean that we only care if the fingers are up or down, yes or no, no partials, no half-way, no angles, no twists. It's meant to illustrate that not only must the storage of your state be a composite but that the representation & manipulation of the information is also extremely complex and must be highly ordered.

Anonymous said...

Jason Long, I probably should've been clearer. Stokes affect a person's thinking, as does a hammer blow to the head. Thus we see clearly that things that affect the grey matter of the brain also affect the "mind" (the theological counter-part is called the "soul"). This is evidence that disconfirms the existence of the soul. If there is a soul it would seem that blows to the head would not necessarily affect one's thinking. Why should they if the soul is not a material entity? This doesn't prove there is no soul, but we cannot disprove much of any such entity, as I've said.

Cheers.

Darrin said...

>>Charlie

//People who go around claiming that theism can't be rational are just victims of Dawkma.//

*Applause*

I think both John and I, and many other users of this blog, will ultimately agree with you on this.

Darrin said...

>>Charlie

//I've shown that to undercut Dawkins's argument one just needs the possibility of a non-mereologically complex object that lacks any parts (save for a single improper part: itself). I gave a thought experiment in favor of this possibility. Even if you don't believe properties exist, nothing hangs on it. As long as the object in my counterexample is complex "in any way" then it's sufficient to undercut Dawkins's premise; this is fairly easy to do without properties, but I'll leave it to the reader to construct her own.
//

A supercellular thunderstorm is a perfect example of what you're talking about, I think. All of the pieces (anvil, wall cloud, tornado, updraft base, etc.) are, metaphysically, comprised of roughly the same components, just in a different relative location. However, each piece of the supercell has its own identity governed by its specific locale in relation to specific shear, wind speed, atmospheric height and temperature, etc., so I am not sure if it will 100 percent suffice.

Adrian said...

A supercellular thunderstorm is a perfect example of what you're talking about, I think.

If so, then it just strengthens the anti-theist case. A thunderstorm and any other weather pattern is composed of trillions of trillions of individual particles, each one of which was pre-existing.

Are you saying that you think God is composed of simpler, pre-existing elements?

Unknown said...

Once someone factually proves to me that unicorns, fairies, all other mythological creatures, and all ancient dead gods that have been dreamed up during the past 10,000 years of recorded human civilization do not exist, I'll be happy to entertain the idea that the Judeo-Christian god exists and is the only true god.

So will roughly 4 billion other non-Christians who currently exist on this planet ;)

District Supt. Harvey Burnett said...

First of all just as the atheist begins with the presuppositional premise that God does not exist, the Christian by EXPERIENCE begins with the presuppositional premise that God DOES exist.

Allowing the premise and basis for each other's assumptions, it is highly reasonable for the theist to ask the atheist, "where's your evidence against my premise?"

As an atheist you have asked the Christian that for years. If a negative or myth can't be proved then why do you ask to disprove it from the beginning?

Simple question: prove why you disbelieve against the same type of evidence that John holds dear;

1-archaeological evidence in favor of a world-wide and biblical flood along with records left in many civilizations describing such, the 2-countless lives that have been changed, lifted from suffering and the burden of sin as a result of experiencing the Christian God,
3-Archaeological evidence that displays a relatively young Earth,
4-Absence of any macroevolutionary record and cross-species speciation.
5-design of complexity pointing toward an intelligent and ordered creation as consistent with the biblical Genesis account of creation.
6-Processes in accord and not in violation of the second law of thermodynamics
7-evidences, testimonies and experiences of countless individuals such as myself who were not raised to be Christians
8- evidences that the immaterial soul exists and no natural or physical evidences, including that found within the genome, that adequately explain and describe the immaterial aspects of man such as love, emotions, desire and ambition to name a few.
9-Finally there is clear evidence that we live in an open continuum which is subject to the intervention of God through miracles supported by millions of testimonies and personal accounts each year.

These are merely SOME of the reasons why I believe and yes these do mirror the reasons that John stated in the article why he doesn't believe.

Beauty is in the eye of the beholder isn't it?

Peace.

Anonymous said...

Hi John

You wrote: "Stokes affect a person's thinking, as does a hammer blow to the head. Thus we see clearly that things that affect the grey matter of the brain also affect the "mind" (the theological counter-part is called the "soul"). This is evidence that disconfirms the existence of the soul. If there is a soul it would seem that blows to the head would not necessarily affect one's thinking. Why should they if the soul is not a material entity?"

I don't think this would disconfirm a Thomistic conception of the soul (in which the soul is the form of the body, and thus necessarily immaterial, given realism; in Thomism, the body and soul form an irreducible unit, but only in the sense that you cannot have a body without a soul; the contrary isn't true).

I think Thomism is being given a bad rap with respect to the notion of god's 'simplicity.' Aquinas didn't simply 'assert' that god is simple (anyone who has read Aquinas knows that he takes very little for granted!); rather, he argued rigorously and at great length, over literally hundreds of pages (in his various works) that god must be simple.

Anthony said...

Harvey,

You crack me up, still beating the same drum huh. Still holding to your flood geology and young earth creationism, still trying to make science fit the Genesis account of history. You still think the silly argument that the second law of thermodynamics refutes evolution. I really wish you would be honest with the evidence so you could see how ridiculous these views are.

So the Christian begins with her presupposition "by experience" of the existence of God. Can you hear yourself? Think man.

District Supt. Harvey Burnett said...

Amp aka Anthony,

I am rubber you are glue whatever you say bounces off of me and stick to U---LOL!

Everything you said about my observations I feel the same about yours my friend...I don't intend to belabor the point but I ask, when will YOU (representative of atheism in general) awake and deal honestly with the EVIDENCE also.

That's the question. Can WE deal honestly with the evidence?

We know that there will be and are some biases, BUT one thing atheist's say that's a TOTAL lie is that "there is no evidence"...WHAT world do you live in to say that? There is evidence, but it doesn't prove what YOU want it to prove, BUT it can't be discounted or thrown off the table just because you don't like it...

So I crack up along with you my friend, because it really is funny!

PEACE.

Anthony said...

Harvey,

Thanks for the good laugh. I really do appreciate your humor.

On a more serious note, you ask:

when will YOU (representative of atheism in general) awake and deal honestly with the EVIDENCE also

I wouldn't say that I am a "representative of atheism in general" although I do strongly suspect that atheism in probably true. I am however open to the evidence and where it leads. As a creationist you do not have that luxury.

Here is the difference between you and me. I have dealt seriously with the evidence. As I have stated before I was a young-earth creationist and flood geologist most of my life. I have read the ICR and AiG books, I used to get all of their newsletters. I embraced the intelligent design movement in the mid nineties and have read many of their books and articles. But what I discovered is that

1) the evidence for an ancient earth and universe is overwhelming

2) the evidence for biological evolution is also overwhelming

3) the evidence of history and historical criticism undermines much of the Bible

These things finally led to a crisis of faith that ultimately resulted of my rejecting Christianity. And what is interesting is that it was the writings of evangelical Christians trying to be honest with the evidence of evolution and historical criticism that ultimately brought me to that crisis of faith.

Darrin said...

//Are you saying that you think God is composed of simpler, pre-existing elements?//

No, I am actually nontheist. I was simply helping Charlie out with a request. ;)

The supercell thunderstorm is an example I often use when theists request of me an example of a sudden appearance of a highly ordered structure from disorder. I don't think it gets props enough.

Larry Hamelin said...

Charlie:

Not so, for the simple reason that it's possible for me to not notice an existing comment by Albert Einstein.

Absence of evidence is not evidence of absence. (Kind of a "duh" issue here.)


You're thinking by slogan, not reason.

Absence of evidence is evidence of absence. Absence of evidence is not certain proof of absence.

In precisely the same sense, presence of evidence is evidence of presence, but not certain proof of presence. You might be hallucinating this message.

Evidentiary reasoning is never certain; any skeptic and naturalist must metaphysically "bite the bullet" and abandon the notion of certainty.

No atheistic naturalist can honestly say that we're absolutely certain no god exists. We can say that we're just as confident that no God exists as we are confident that the law of gravity is true.

Larry Hamelin said...

Harvey:

First of all just as the atheist begins with the presuppositional premise that God does not exist...

Not true. Please don't speak to what atheists do and do not believe in general without supporting evidence.

the Christian by EXPERIENCE begins with the presuppositional premise that God DOES exist.

If this is the case, then evidence is irrelevant. Evidence can neither confirm nor deny a presupposition.

Allowing the premise and basis for each other's assumptions, it is highly reasonable for the theist to ask the atheist, "where's your evidence against my premise?"

Well, no. If you're saying that Christians presuppose the existence of a god, then evidence is irrelevant. If you're simply being imprecise and say that Christians hypothesize the existence of a God, then it is certainly reasonable to ask the skeptic for the evidence against.

Of course, to frame the existence of God as a hypothesis, you must draw specific, observable consequences from that hypothesis than can be confirmed or falsified by observation and experience.

Your notions of "evidence" seem... quaint. Let me deal with the first two:

1-archaeological evidence in favor of a world-wide and biblical flood along with records left in many civilizations describing such

2-[the] countless lives that have been changed, lifted from suffering and the burden of sin as a result of experiencing the Christian God,


We have to distinguish between the fact entered into evidence, and the conclusions drawn from those facts. In both of these examples, you talk about the conclusions, not the facts.

In 1) "evidence in favor of a world-wide and biblical flood," The conclusion follows "in favor of". You present no actual facts.

In two, you talk about "lives that have been changed... as a result of experiencing the Christian God." Again, your conclusion follows "as a result of". You allude to some facts, but don't present them: Whose lives have been changed? In what way?

Larry Hamelin said...

Charlie: The phrase "absence of evidence" is equivocal. It can mean, "I didn't look," or it can mean, "I looked, but I didn't see X".

Obviously in the first sense, absence of evidence is not even evidence of absence. The world doesn't go away when I close my eyes.

The second sense is more subtle. "I looked but I didn't see X" is not really absence of evidence; it can be restated as, "I looked and I saw Y", which is the presence of evidence. If Y is incompatible with X, and the presence of Z predicts X and not Y, then we have evidence of absence.

It is merely exploiting an ambiguity in English grammar to label "I looked but didn't see X" as absence of evidence.

David B. Ellis said...


Whatever their respective truth-values turn out to be, any serious participant in these debates would acknowledge that theism and atheism are both rational points of view, held by many brilliant philosophers and scientists on each side of the division.


One is not obligated to concede that the opposing position's view is rational.

The fact that many philosophers and scientists have believed in it has nothing to do with whether its rational when none of those philosophers or scientists have good arguments for their position.

One has little trouble finding philosophers and scientists defending baseless beliefs of the most fringe imaginable pseudo-science and supernaturalism. It should be no surprise that we can find even more of them defending mainstream baseless beliefs like theism and christianity.

Philosophers and scientists are as fallibly human. I'm interested in the actual arguments ---not appeals to authority.

David B. Ellis said...
This comment has been removed by the author.
David B. Ellis said...


First of all just as the atheist begins with the presuppositional premise that God does not exist, ....


I am an atheist and begin with no such presupposition.


That you are a presuppositionalist does not mean that the opposing side shares that epistemology---neither in theory nor in practice.

David B. Ellis said...


Allowing the premise and basis for each other's assumptions, it is highly reasonable for the theist to ask the atheist, "where's your evidence against my premise?"


If one holds a presupposition which is mistaken all that follows in your thinking will reflect that fundamental error.

Its precisely in one's most basic premises that one should be the most careful.

So the correct response to your question is to point out that nothing on which empirical evidence can be relevent in deciding whether its true should be taken as a presupposition. To do so just opens one to error.

One should not take positions on the existence or nonexistence of particular things, be they a god, gods, angels, demons, fairies, planets, galaxies, atoms, or anything else, as basic presuppositions.

The sensible person who is interested in the truth rather than clinging to a belief system only takes things as presuppositions which are truly basic and self-evident.

Like laws of logic and mathematical truths. To do otherwise is to set oneself up for error of a truly global scope.

Jeff C said...

One should not take positions on the existence or nonexistence of particular things, be they a god, gods, angels, demons, fairies, planets, galaxies, atoms, or anything else, as basic presuppositions.

The sensible person who is interested in the truth rather than clinging to a belief system only takes things as presuppositions which are truly basic and self-evident.


Thank you. I get so tired of hearing about "presuppositions." That concept does not exist in the world of science, because the moment you introduce such a concept is the moment you're leaving science and entering the world of the imagination.

Your mentioning of atoms is perfect. Physicists and chemists didn't start with a "presupposition" that atoms exist and build their theories on that foundation. (Indeed I still occasionally hear people say "we don't *really* know if atoms exist." In fact, we know they exist; it is a fact.)

The only "presupposition" that exists in science is that we don't know anything and that we will learn by discovery. Nothing more. The only people harping on "presuppositions" are preachers and theologians who feel the need to somehow defend their absurd notion that they'll start with the Bible as a fundamental truth -- an approach that holds no place in science. And so they introduce this silly word "presupposition" and try to put themselves on equal footing as the scientists by claiming that they just have different "presuppositions."

Even mathematics is not a "presupposition". My bachelor's degree is in mathematics with minors in physics and computer science. During courses we actually take a step back from all the calculus and more advanced mathematics and explore the properties of numbers themselves. This is where we discover things like *why* a negative number times a negative number makes a positive number. We don't just assume it to be true and call it something nonsensical like a "presupposition". Rather, we explore and discover the truths of how and why it happens. Even things like complex numbers have a how and why and can be demonstrated realistically through electrodynamics and quantum mechanics.

And while on this topic, something I've seen is theologians argue that logic is the realm of philosophy and not of science and they try to act like they know more than we do about it. Here they once again have it backwards. Mathematics is deeply rooted in logic. During undergrad, I had an entire year of logic courses that were almost identical to those found in the philosophy department. We studied symbolic logic, and this is fundamental part of mathematics, because mathematics relies on logical proofs. From about second year on, you will not find a mathematical course that doesn't conduct logical and mathematical proofs. And from mathematics you can obtain the foundation for physics and chemistry, which in turn lay the groundwork for biology and other sciences.

And so all the theologians are doing is demonstrating their lack of understanding of science. Logic is indeed a part of science and has mathematical rigor and experimentation to back it up. "Presupposition," on the other hand, has no place in science.

So people can imagine angels and demons and gods and devils in their heads, and back it up with a supposed presupposition and then try to logically build up arguments based on such nonsense, but their arguments have no bearing whatsoever in reality.

(In my next rant I'll talk about, having studied graduate-level physics and having had an entire course in thermodynamics, how annoying it is to hear theologians with no scientific training whatsoever make absurd, ridiculous claims about certain areas of science supposedly violating the Second Law of Thermodynamics, a topic they know absolutely nothing about.)

Anonymous said...

Hi Jeffery

You wrote: "I get so tired of hearing about "presuppositions." That concept does not exist in the world of science, because the moment you introduce such a concept is the moment you're leaving science and entering the world of the imagination."

I will just note that the idea of a presuppositionless discipline is logically impossible. You can neither justify every proposition
nor demand justification for every proposition: in either case, you're facing an inescapable logical regress.

Note also, the 'only presupposition' you claim science entertains, viz. "we don't know anything and that we will learn by discovery," *itself* is overflowing with presuppositions!

You wrote: "The only people harping on "presuppositions" are preachers and theologians who feel the need to somehow defend their absurd notion that they'll start with the Bible as a fundamental truth -- an approach that holds no place in science. And so they introduce this silly word "presupposition" and try to put themselves on equal footing as the scientists by claiming that they just have different "presuppositions.""

This is simply not true. Philosophers pay a great deal of attention to presuppositions. If you want to try to identify 'philosophers' with 'preachers and theologians,' you'll be ignoring the fact that you're *much* more likely to find an atheist in the philosophy department than you are among scientists.

kiwi said...

So has anyone proposed a "model" of how God manages to be omniscient, omnipotent while being simple? How does he manage the vast amount of information about the universe? Plantinga says God is a "spirit". What is a spirit made of?

David B. Ellis said...


I will just note that the idea of a presuppositionless discipline is logically impossible. You can neither justify every proposition
nor demand justification for every proposition: in either case, you're facing an inescapable logical regress.


I have no particular problem with taking a few thinks as "givens". Things that are simply self-evident. Mostly here I'm referring to logical truths.

I don't agree, though, that we are obligated to have metaphysical presuppositions---to make assumptions about the nature of reality.

Instead I think what most of us naturalists (and what good science does) is observe the world and ask themselves, "what model fits best with these observations". There is no infinite regress in doing this.

Doing so does not, it seems to me, require that I start with any metaphysical presuppositions.

If you disagree, and think I hold unrecognized metaphysical assumptions I invite you to state what you think they might be (and argue for why I must be holding such presuppositions rather than following the alternative approach briefly outlined above).

Anonymous said...

Hi David

You wrote: "I don't agree, though, that we are obligated to have metaphysical presuppositions---to make assumptions about the nature of reality.
Instead I think what most of us naturalists (and what good science does) is observe the world and ask themselves, "what model fits best with these observations". There is no infinite regress in doing this.
Doing so does not, it seems to me, require that I start with any metaphysical presuppositions."

I would begin with a distinction between 'naturalism' and 'science.' Naturalism is by definition a philosophic take on the nature of the world, whereas science (understood strictly!) is in essence a particular way of investigating the world, *regardless of it's nature*. Further, I would point out the fact that science in no way supports naturalism (e.g. modern science cannot tell you if naturalism is true and, say, Berkeley's idealism false: the results of modern science would be *exactly the same* if *either* obtained). In this sense, I don't see how you can claim that naturalism doesn't commit one to metaphysical presuppositions.

Further, you claim that the naturalist and the scientist "observe the world and ask themselves, "what model fits best with these observations". It seems to me that you are indeed knee-deep in metaphysical assumptions here as well: there is a world that exists independently of the observer; the world is orderly and can be understood by human cognition; matter has not just a chronological primacy over consciousness, but an ontological primacy over consciousness; and so on. Of course, you're also committed to a number of related epistemic presuppositions that cannot be reduced to 'logical truths,' e.g. our senses and cognition can accurately perceive and conceive reality, our memories are reliable, etc.

If you start with such presuppositions, then you're indeed not facing an infinite regress; however, if you demand a presuppositionless approach to understanding the world, whether philosophically, scientifically, etc., then you are indeed in the grips of an inescapable regress, one that an assent to the presuppositonal nature of fundamental logical truths alone cannot curtail (it's hard to claim that the law of non-contradiction obtains without committing yourself to a host of logically anterior propositions, e.g. something exists, I can understand it, etc.).

a helmet said...

There is no evidence for extraterrestrials either


remarks a helmet

Anonymous said...

Let me clarify one point: You can't get to 'naturalism' by way of fundamental observation, a previous commitment to some fundamental logical truths, and science alone. Again, our observations, logical truths, and scientific discoveries would be *exactly* the same given: naturalism; Berkeley's idealism; Kant's transcendental idealism; *or even classical theism*.

a helmet said...

mathyoo,


All of those [gods] either have or have had believers just as ardent as any Christian, yet there is exactly as much evidence for the existence of any of them as there is for the Christian God. That is, to say, no evidence at all. Yet Dr. Craig, as well as pretty much every other Christian on the planet, would deny the existence of those other Gods but not their own.

There is a difference between the Christian God and those you enumerate. The christian God was revealed through a historical figure, Jesus Christ. This historical figure performed miracles that were witnessed by many. Ultimately he was raised from the dead.

Now, there can be much discussion about "God" who/what he might be, what he looks like, what he wishes, what he's able to, his character and so on and so forth. There can be never ending discussions about that. But there can be no such questions about historical persons. A historical figure is not a matter of guess.


Then there's the issue of the definition of the Christian God. In order to be able establish the existence of any given entity, you have to be able to define that entity somehow. If the definition is logically incoherent, as most Christian definitions of their God seem to be, it's not any more difficult to disprove God than it is a square circle.


There is much theological disunity to be sure. But the non-christian gods don't have any basis in reality. No miracles by Zeus have been reported, no one ever said if you have seen me, you have seen Allah! , no one saw a resurrected Hindu god etc.

While theological unity is far away, the historical settings are firm.

Only christianity is based on solid historical records. And that is the difference to all the other beliefs. So there is an interface between the christian God and the world, provided by the historical Jesus, whereas such an interface is lacking in other religions.


Charlie, per your first comment, we'd also expect to see a God if it existed. Yet, we don't. We only have a few very questionable, unverifiable accounts by people claiming to have spoken to God.


I disagree. The miracles and the resurrection of Jesus were testified by hundreds of persons, even hostile ones.

Philip R Kreyche said...

Helmet,

Who are you to say that no other gods in history have had any basis in reality? That's an extremely prejudiced view of things. The Greeks, Romans and Gauls divined the will of the gods through divination; Asclepius was said to heal those who came to his shrines; Zarathustra claimed to have directly experienced a revelation from Ahura Mazda; shamans regularly claim direct contact with the gods; in Tibet, shamans even today prove that the gods have blessed them and talked to them by performing miraculous deeds that people cannot explain; and of course Muhammad also claimed contact with God, and this was based in reality.

Religions the world over have consistently based their beliefs on the contact the gods have with the world. Why else would people have followed them, if they didnt feel there was a reason? For you to claim (without evidence) that no religion but Christianity has been based in reality is arrogant and colossally misinformed.



(Oh, and name one person "hostile" to Christianity that independently attested to Jesus' resurrection)

Philip R Kreyche said...

And also please provide links to the "hundreds" of different, independent eyewitness testimonies for Jesus' resurrection.

Adrian said...

The gospels say that hundreds of people saw Jesus rise from the dead. What more do you want? You sceptics are never satisfied!

:)

a helmet said...

Oh, and name one person "hostile" to Christianity that independently attested to Jesus' resurrection

Saul-Paul.


Tyro, not the gospel accounts say so, but the letter to the Corinthians, written ca. 50-55 A.D.

David B. Ellis said...


I would begin with a distinction between 'naturalism' and 'science.'


Agreed. That's why I referred to them as separate things.


Further, I would point out the fact that science in no way supports naturalism....


Sure it does---it just doesn't prove naturalism (which would be impossible). In all the centuries we've been doing science all the things its investigated and which were thought to be supernatural have turned out to have naturalistic explanations (at least all the things we've so far discovered the explanation for).

Things could have turned out differently (assuming we live in a world that includes the supernatural). We could have found wizened Hindu yogis living in the mountains who have the ability to levitate. We could have found hard evidence that nothing in our cosmos is more than 6000 years old. We could have been able to document, repeatedly, with scientific devices the existence of ghosts to such a degree that no rational person could fail to be convinced.

But we don't.

And, no, this isn't disproof of the supernatural (there is always the bare possibility that we will find good verifiable evidence for it in the future). But it IS good reason to refrain from believing in the supernatural (which is all that my own naturalism amounts to---the absence of belief in supernatural phenomena until such time as good reason to be convinced is found).

.g. modern science cannot tell you if naturalism is true and, say, Berkeley's idealism false: the results of modern science would be *exactly the same* if *either* obtained...


Don't make the mistake of equating naturalism with materialism. If the ultimate "stuff" of which reality is composed behaves exactly the same way it doesn't make an iota of difference if you call it "idea" or "matter"---there is no substantive difference.

In such a case the materialism/idealism debate is just semantics.


It seems to me that you are indeed knee-deep in metaphysical assumptions here as well: there is a world that exists independently of the observer.....


I do not presuppose this.


.....the world is orderly and can be understood by human cognition.....


Why should I start from the presupposition that the world is orderly? I observe that the world of my sensory experience is orderly and manage to make predictions within it which would not be possible if there was no order to be found.


matter has not just a chronological primacy over consciousness, but an ontological primacy over consciousness;



I hold no such presupposition.


and so on. Of course, you're also committed to a number of related epistemic presuppositions that cannot be reduced to 'logical truths,' e.g. our senses and cognition can accurately perceive and conceive reality, our memories are reliable, etc.


Perhaps, though only tentatively. And I didn't claim that there are no legitimate epistemic givens.

I said there is no need for metaphysical givens. I gave laws of logic as the most obvious example of givens I can accept---not the only ones.

As to the two epistemic presuppositions you propose, I, in fact, don't agree with them.


our senses and cognition can accurately perceive and conceive reality


I do not think our senses are accurate in their perception of reality. We often, in fact, misperceive. I don't assume that our senses contact "reality" at all, for that matter.

For all I can know, we might be living in a computer simulation.

But I have no particular reason to think such a thing is the case and my perceptual universe is the only one I have to inhabit---so its the one I deal with. There is no way to prove or disprove "brain in a vat" scenarios and no particular reason to bother trying.


our memories are reliable


On the contrary, I certainly don't hold THAT presupposition. Our memory appears to be terribly fallible.

Of course, if both my memory and my perception is in total error then I can never know anything at all.

I have no particular reason, though, to think they are. I deal with the sensory perceptions and memories I have. This does not involve an assumption of their reliability (in fact, I think they are likely to be in error fairly often)---only a recognition that there is no workable alternative to doing so.

Adrian said...

Helmet,

So you have one friendly source, even if he was once hostile, and Paul never saw Jesus except as a vision so he certainly couldn't offer testimony. Still waiting for "[testimony] by hundreds of persons, even hostile ones."

David B. Ellis said...


There is no evidence for extraterrestrials either


remarks a helmet



Which is why I'm not a believer in extraterrestrials.

Their existence awaits confirmation. Until then, all that can be said is that we don't know.

I do, though, think its more likely than not that they exist. We know intelligent beings can evolve on planets. We have a massive cosmos with many opportunities for it to happen.

If I had a million dollars riding on the question I'd put my money on their existence. But, even so, that's just what I consider the best bet---I don't know one way or the other.

David B. Ellis said...


No miracles by Zeus have been reported, no one ever said if you have seen me, you have seen Allah! , no one saw a resurrected Hindu god etc.


If you don't think Muslims, Pagans and Hindus claim miraculous events verify their religion then you just haven't read enough on those topics. The miracles of your religion are unsupported by strong evidence that they actually occurred---same as theirs.

a helmet said...

Tyro, 1 Cor. 15 ff. is the source. The letter fulfills all requirements of historical credibility one could reasonably ask for.


Note, Saul was accompanied by at least 2 men, who also heard Jesus' voice. So the encouter was not merely subjective.

David B. Ellis said...


Oh, and name one person "hostile" to Christianity that independently attested to Jesus' resurrection

Saul-Paul.



Except, of course, that Paul did not witness the resurrection. He merely had a vision on the road to Damascus.

He can "attest" all he likes that the vision was of a really resurrected Jesus rather than a hallucination---it's still about as weak, evidentially, as can be imagined.

Unknown said...

A Helmet:

There is no evidence that the historical person named Jesus that is described in the bible existed at all. Although it is reasonable to assume that there were many hundreds, if not thousands, of people named Jesus wandering around the ancient world.

There is absolutely no evidence at all, other than claims made in the self-validating bible, that any miracles were performed by anyone, or that anyone rose from the dead. We do, however, have lots of evidence that the same miracles allegedly performed by the biblical Jesus, including rising from the dead, had been attributed to a vast array of mystery cult figures and pagan gods centuries before the story of Jesus was written/plagiarized.

By the way, you would most likely have been killed many times over for doubting the literal existence and divinity of hundreds of ancient dead gods depending on when and where you lived in ancient times. The people who believed in the ancient gods took their gods just as seriously (if not more seriously) as the most devout evangelical Christian takes theirs today.

David B. Ellis said...


Note, Saul was accompanied by at least 2 men, who also heard Jesus' voice. So the encouter was not merely subjective.


Correct me if I'm wrong, I haven't checked yet, but arent there two biblical accounts of Paul's vision? In one of which it is said that his companions heard the voice and in another that they didn't.

And even if I'm mistaken about that---we only have his word that someone else heard the voices---hardly strong evidence. We don't exactly have his companions account of the events.

a helmet said...

David B. Ellis,

The miracles of your religion are unsupported by strong evidence that they actually occurred

Jesus' miracles are mentioned in the Talmud. Though they are interpreded as sorcery there, this is still evidence that they happened. A hostile confirmation is a strong one, even if evaluated differently. Josephus mentions Jesus' miracles as well.


Regarding Mohammed, the only miracle he claimed to have offered was the Qur'an. "I have given you a book" are supposedly Mohammed's words at the end of his life.

Unknown said...

Jesus' miracles are mentioned in the Talmud.
Um...wasn't the modern Talmud composed several hundred years after Jesus supposedly lived? It's easy to prognosticate after the fact.

Josephus mentions Jesus' miracles as well.
This has been widely regarded as a fabrication for a long time.

Regarding Mohammed, the only miracle he claimed to have offered was the Qur'an.
So the whole flying up to heaven on a winged horse thing doesn't count?

a helmet said...

David B. Ellis,

There are 3 passages -- all from the same book, namely Acts, to consider. Without an in-depth explanation here, Jesus talked to Saul in Hebrew at the road to Damascus. However, the men that accompanied Saul, did not understand Hebrew, so they heard the voice but did not "hear", did not understand. Because of the foreign language.

And the men which journeyed with him stood speechless, hearing a voice, but seeing no man. Acts 9:7



And they that were with me saw indeed the light, and were afraid; but they heard not the voice of him that spake to me.
Acts 22:9


And when we were all fallen to the earth, I heard a voice speaking unto me, and saying in the Hebrew tongue, Saul, Saul, why persecutest thou me? it is hard for thee to kick against the pricks.
Acts 26:14


So these men nevertheless became witnesses of a physical appearance.

Adrian said...

Helmet,

We've gotten very far off topic so I'll be brief. You've claimed that we have hundreds of testimonies but we have a single account by a friendly source which claims that others were witnesses. Is it really your contention that this is sufficient to establish that a miracle occurred? Be careful when answering, there are many other non-Christian miracle claims which have more evidence than this.

Philip R Kreyche said...

I thought Helmet was joking, but apparently he isn't. He's obviously way out of his league since he knows not even the basics of the religions he's atracking. And he thinks that one stray unsupported claim in a piece of religious propoganda of the existence of hundreds of anonymous unverifiable eyewitnesses fits the highest standards of historical inquiry.

Maybe we should just leave him alone.

Adrian said...

I agree. Is there any other comments regarding evidence against God?

Charlie, have you got any reason to think that "spirit" can be both capable of thought, knowledge and action yet not be composed of other, simpler parts?

Russ said...

District Supt. Harvey Burnett,

To repeat what I said in the post titled "The Bizzaro Beliefs of Christianity" in response to your specious comment about orthodoxy and orthopraxy, what you are saying here is also smokescreen pure and simple. You are an obscurantist charlatan, Harvey.

Your claim,

First of all just as the atheist begins with the presuppositional premise that God does not exist, the Christian by EXPERIENCE begins with the presuppositional premise that God DOES exist.

is truly dishonorable, Harvey. As I pointed out in the above mentioned comment, if the fundamentalist Christians being guided by your supposed divine insight actually compared their own life circumstances to the rest of those sharing their Christian label in the US, they would realize that their EXPERIENCE is of significantly lower quality than what it could be. If all US Christians actually compared their lives to those of the many European countries where the fog of Christendom is largely an historical relic, they too would find things wanting.

Your claims, Harvey, that belief makes life better for your followers, is not born out in any reasonable measures of personal or social well-being. What people have the lowest infant mortality rates? Not the US god gloaters. It's the people living where religion plays a ceremonial afterthought sort of role, places like Japan, Sweden, Australia and Denmark. Those people lose their newborn babies the least where the people themselves take the responsibility for the welfare of those infants rather than paying someone like you, Harvey, to callously explain away those lost lives as part of some incomprehensible mystery called God's Divine Plan.

Is infant mortality the only such objective measure where US Christian EXPERIENCE, especially US fundamentalist Christian EXPERIENCE is far less comforting and supportive of their dishonest clergy's claims than is the experience of non-believers? Not by a long shot. In measures of violent crime -including rape, incest, armed robbery, murder, and domestic abuse - divorce, abortion, teen pregnancy, preventable health problems like obesity, educational attainment, income and access to healthcare, fundamentalist Christians fare worse than non-believers.

Now before you start convulsing, Harvey, that these are atheist or liberal studies, let me tell you they are not. Most are simple rates determined from government gathered data on health and human welfare. Others are studies done by groups like the Barna group, themselves Christians, interested in how the quality of Christian life compares to the lives of non-Christians and non-believers. They demonstrate that for all the prayers superstitiously prayed and for all the money they've put into the pockets of clergy, they are not as well off as those who don't pray to the imaginary and who don't pay clergy for expensive and ineffective wishful thinking.

I know you are aware of where your congregation stands regarding these very human concerns, Harvey, since you list many of them on the homepage for your church. But, as with most Christian clergy, you cannot follow the morally correct path of counseling your congregation to invest their money in things that would brighten the future prospects for themselves and their families like education, healthcare, and physical fitness. No, instead, you tell them fairy tales about some imaginary deity giving them a ten-fold return on their investment in your business enterprise. That is, the gospel of your own personal interests keeps them from looking out for their best interests.

If Christians were sufficiently concerned with their EXPERIENCE they would stop heeding your advice, stop paying your bills, and stop wasting their money on your demonstrably pointless venture. The world they could EXPERIENCE beyond your church sanctuary, if their money worked for them instead of for you, could be happier, brighter and more at peace.

Your disservice to your congregation extends beyond your self-serving ancient fables when you lie to them about the findings of modern science - another of your clerical strategies of promoting ignorance in your congregation to pad your pocket. The claims you make in your comment are lies, Harvey, pure and simple. You lie about archeology; you lie about evolution; you lie about physics when you mention the second law of thermodynamics; and, you even lie when you suggest that personal testimony constitutes proof of your version of a god. Anyone, me, for instance, who has studied these things, knows beyond all doubt that you have not. You've made no honest effort to understand any of these matters. You have not studied archeology and evolution and physics and found the evidence for their accepted claims wanting. So, one can only assume you propagate ignorance you gleaned from someone else, and you preach it as a fifth gospel to poison your congregation's attitude toward the science that continually enriches their lives. Tragically, your ignorance also stamps out much of the natural curiosity about science that the children in your congregation would otherwise have, leaving the good paying and well respected careers in science out of their considerations about their futures. It is inherently cruel and heartless as well as morally corrupt for a clergyman to intentionally instill ignorance of and disdain for science - a thing on which all our lives depend, mind you - in a child, to assure that, as that child grows into a wage earning adult, their inculcated distrust of science will keep the offering plates full and the clergyman's financial prospects bright.

You said, Harvey,


These are merely SOME of the reasons why I believe and yes these do mirror the reasons that John stated in the article why he doesn't believe.

Beauty is in the eye of the beholder isn't it?


Your twisted takes on reason, logic, and science are not a case of beauty being in the eye of the beholder. If it was science could never have made the many advancements that have so benefitted mankind. If the causes of reason, logic, and science were quite as your comments so often reflect, we would still be torturing people for demon possession, bloodletting to let out the evil spirits, and putting in chains those suffering from diabetes, dementia, epilepsy, autism, and mental and physical handicaps, exactly as your clerical predecessors prescribed. Your version of god didn't protect those completely innocent, but nonetheless brutalized, victims of the past, and, today, it's not protecting your congregation from the psychological and intellectual victimization meted out by you.

David B. Ellis said...


However, the men that accompanied Saul, did not understand Hebrew, so they heard the voice but did not "hear", did not understand. Because of the foreign language.


Sounds like inventive reinterpretion to me. But let's be generous and suppose for the sake of discussion that where it says "they heard not" it would have been more accurately translated "they understood not".

Its still just a single document making this claim. One about which scholars continue to dispute the authorship.


Regarding Mohammed, the only miracle he claimed to have offered was the Qur'an. "I have given you a book" are supposedly Mohammed's words at the end of his life.


Muslims claim (as many christians do concerning the bible) that the Koran anticipates future scientific discoveries which show that it could not be a merely human document. The claims are absurd in both cases though.

And I am referring here, as much to contemporary miracle claims, as to ancient ones. Hindus claim to have seen yogis perform all manner of supernatural feats with their own eyes. They claim evidence of knowledge of past lives and other miracles.

Surely this amounts to far more than some ancient documents, mostly of uncertain authorship, written, at best, decades after the events.

And yet you are unconvinced by contemporary eyewitnesses claims of the supernatural among other religions but think we should credit these ancient documents as hard evidence for christianity?

Methinks I smell a double standard.

Anonymous said...

Eric: "Further, I would point out the fact that science in no way supports naturalism..."

David: "Sure it does---it just doesn't prove naturalism (which would be impossible). In all the centuries we've been doing science all the things its investigated and which were thought to be supernatural have turned out to have naturalistic explanations (at least all the things we've so far discovered the explanation for)."

Though you initially acknowledged the distinction between naturalism and science, here you've confused the two. Science, qua science *cannot* support a metaphysical conclusion (such as naturalism). Science has not confirmed, or even provided an iota of support for, naturalism; rather, naturalists interpret scientific discoveries naturalistically. There's a huge difference there: in the first case, we're talking about methodological naturalism, which science *presupposes* for the sake of inquiry, but which is devoid of ontological commitments; in the second case, we're talking about scientific naturalism, which *does* make the sorts of (scientifically unjustified) ontological commitments that methodological naturalism avoids. For example, does evolutionary biology support naturalism because it has effectively ruled out traditional creation myths? Absolutely not: it in no way precludes or provides evidence against a layered approach to understanding reality, e.g. one which embraces both mechanism and teleology. (Most atheist appeal to Ockham's razor here, but in doing so they demonstrate that they don't understand it: Ockham's razor is used to choose among *competing* *explanations* of a scientific or quasi-scientific nature; in my example, however, teleology doesn't compete with mechanism at all, and it isn't an instance of a scientific or quasi-scientific explanation, but of a metaphysical claim.) In short, it in no way follows from the fact that science has falsified *competing* supernatural explanations that science has therefore provided evidence against the supernatural as such (just as it would not be the case that if we were to falsify every claim of alien abduction we would have therefore provided evidence against the proposition, "Extraterrestrial life exists"; rather, we would only be justified in concluding that aliens aren't abducting people).

David: "Don't make the mistake of equating naturalism with materialism. If the ultimate "stuff" of which reality is composed behaves exactly the same way it doesn't make an iota of difference if you call it "idea" or "matter"---there is no substantive difference."

First, what we call it isn't the issue; what it actually *is* is the issue. While the difference may not make a difference *scientifically*, it *does* make a difference metaphysically. Note, however, that by conceding that this metaphysical difference makes no scientific difference, you've conceded that science can say nothing about these sorts of metaphysical conclusions, and thus that science cannot support naturalism, however one conceives it! And you can't respond by claiming to deny metaphysics altogether; it's been demonstrated time and time again that this is *itself* a metaphysical conclusion, with a plethora of metaphysical presuppositions and implications.

Second, however, I'll certainly grant that there are sundry variations on the 'naturalism' theme, but most naturalists today are materialists (actually, physicalists, which can be distinguished from materialism)because they accept the causal closure thesis (which entails physicalism). If you accept the causal closure thesis, and therefore physicalism, my initial criticism stands (which, given the distinction I made above between what we call something and what it is, and the implications of this distinction for your position, puts you in a tough place). If you reject the causal closure thesis, and materialism, then your naturalism reduces to the sort Plantinga speaks about, namely the rejection of the supernatural. However, this puts you in an impossible position, since science cannot in any way provide support for the proposition, "There is no supernatural." At best, science can provide explanations that rule out *competing* supernatural explanations; it can say *nothing*, however, about the sorts of layered, non-competing metaphysical deductions (not 'explanations'!) I referred to earlier. So, if this is the sort of naturalism you subscribe to, your claim that science supports it is untenable. Also, a fortiori, we have a defeater for your claim that the sort of naturalism you subscribe to is without presuppositions.

Eric: "there is a world that exists independently of the observer....."
David: "I do not presuppose this."

If not, then your claim that naturalists and scientists "observe the world and ask themselves, 'what model fits best with these observations'" is incoherent.

Eric: "the world is orderly and can be understood by human cognition..."
David: "Why should I start from the presupposition that the world is orderly? I observe that the world of my sensory experience is orderly and manage to make predictions within it which would not be possible if there was no order to be found."

This too is incoherent. If you don't presuppose that the world is orderly, then you have not only have no reason to trust your predictions -- you have no reason to make them. Not only that, but your present observations of order don't give you a non-question begging ground on which to expect order in the future.

David: "And I didn't claim that there are no legitimate epistemic givens.
I said there is no need for metaphysical givens."

Can you provide an example of an epistemic presupposition that isn't itself committed to at least one metaphysical presupposition?

With respect to your claim to repudiate the reliability of our memories, I'll just point out that it's self-defeating: you had to rely on your memories in order to formulate your repudiation of them!

Finally, your claim that you don't believe that our senses are accurate is inconsistent with your claims above about observing and modelling the world, as well as your claim that you "observe that the world of [your] sensory experience is orderly and manage to make predictions within it which would not be possible if there was no order to be found."

David B. Ellis said...



Though you initially acknowledged the distinction between naturalism and science, here you've confused the two.


No, I haven't. I've simply stated my reasons for thinking that what science has found, throughout its history, fits best with naturalism.

As I explained, different scientific findings could have led to a confirmation of supernaturalism.

If the supernatural does not exist, science cannot do anything but never confirm the existence of the supernatural (exactly what has so far taken place). There are all manner of observations science could have made though (I listed many) which would have conclusively disconfirmed naturalism and confirmed the supernatural.


While the difference may not make a difference *scientifically*, it *does* make a difference metaphysically.


No, it doesn't. If you disagree tell me what the substantive difference is. What makes the fundamental "stuff" of reality if it is called idea different from one where its called matter if there are no difference in the way they work.


Note, however, that by conceding that this metaphysical difference makes no scientific difference, you've conceded that science can say nothing about these sorts of metaphysical conclusions....


I've said nothing of the sort. We were talking about a limited hypothetical case where idea and matter as the basic "stuff" of the world have no observable differences. This is not necessarily the case. One can, for example, imagine a variety of idealism where there ARE observable differences. For example, a version of idealism where mind can act independently of the body and allow for the existence of psychic powers having no physical processes as their basis.

I also gave a list of examples of observations which would serve to confirm supernaturalism.

So obviously I do think science can provide support for metaphysical theories.

Do you deny that supernaturalism could be supported by scientific observations like the ones I mentioned?


you reject the causal closure thesis, and materialism, then your naturalism reduces to the sort Plantinga speaks about, namely the rejection of the supernatural.


Almost. My naturalism consists of the absence of belief in supernatural phenomena---and I am defining supernatural phenomena to be phenomena where mind acts, exists, gathers knowledge, etc without any physical basis for doing so. For example, ghosts, souls and God are supernatural in this regard because they are mind (or spirit, whatever you wish to call it) existing without any physical basis---like a human body with its brain. Magic and psi are supernatural in the sense of being the ability of mind to perform feats like seeing distant events without ones senses and moving objects with ones mind, etc.

I'm sure you get the idea.

In that sense it can be described as a variety of physicalism---and, again, consists simply in the absence of belief in phenomena with no physical basis.

If provided with evidence of such phenomena that absence of belief would be gone.


However, this puts you in an impossible position, since science cannot in any way provide support for the proposition, "There is no supernatural."


It provides support for the proposition "there is no basis for belief in the supernatural". Which is all my variety of naturalism (absence of belief in the supernatural) demands of it.


If not, then your claim that naturalists and scientists "observe the world and ask themselves, 'what model fits best with these observations'" is incoherent.


The sensory world is the only world we have available to us---so its the one I deal with. I think the hypothesis that it reflects an actual external world is the most plausible but don't assume I'm right about this being the case. I'm willing to entertain "brain in a vat" scenarios. They are, after all, unfalsifiable. Therefore, it cannot be said that I hold the presupposition that there is an external world at least somewhat accurately reflected by my senses. At best, I consider it the most sensible working hypothesis (mostly on the grounds of occam's razor).


If you don't presuppose that the world is orderly, then you have not only have no reason to trust your predictions -- you have no reason to make them.


I don't presuppose the world to be orderly. I observe orderly predictable phenomena and act accordingly.


Not only that, but your present observations of order don't give you a non-question begging ground on which to expect order in the future.

If you wish to raise Hume's problem of induction I can answer that with no recourse to presupposing orderliness but its a long discussion in its own right and I'm not that interested in getting sidetracked any more than we already have. To give the nutshell version---if order continues in the future my acting accordingly will be rewarded with the best possible opportunity for success. If order continues but I dont act according to past observations I will most likely die (walking off the roof will kill me? why think that? here I GGGOOOoooooo.......splat). If, however, order ends then I have no reason to think acting according to past observations will yield worse results than any alternative.

So there's no basis for not employing induction (acting on the basis of past observations of the way the world is ordered).

No question-begging circularity involved.


Can you provide an example of an epistemic presupposition that isn't itself committed to at least one metaphysical presupposition?


So far you haven't even pinned me down to any presuppositions of either variety. Once we come up with a epistemic presupposition I hold I'll examine it to see if it requires that I hold some metaphysical presupposition as well.

My only metaphysical opinion so far discovered is a negative variety of naturalism (which consists of an absence of belief in the supernatural). There doesn't seem to be much presuppositional potential in that.


With respect to your claim to repudiate the reliability of our memories, I'll just point out that it's self-defeating: you had to rely on your memories in order to formulate your repudiation of them!


Its hardly self-defeating. My memories include examples of my memory being shown to be mistaken (if only it didn't---I'd have gotten straight 100's in history). So, at best, my memory is only partially reliable.

My approach to this question is the essentially the same as my approach to the problem of induction---if my memory is always wrong then I'm screwed anyway. My only option is to use the memories I have in thinking about the world.

In strict philosophical terms I'm a pretty thoroughgoing skeptic. You're going to have a tough time trying to making me out to be a presuppositionalist in any substantial sense. But feel free to try. Bang your head against that steel wall as much as you like.

David Parker said...

David Ellis,

Do you deny having presuppositions?

David B. Ellis said...

I already said I consider laws of logic to be self-evident givens.

We been searching and I've so far found nothing else. I don't think there cannot be other ideas which I would also think legitimate to take as "givens", "properly basic beliefs" or "presuppositions", whatever you prefer to call them.

I just haven't had much success so far at finding any.

Perhaps you'd like to propose something else for consideration as a legitimate presupposition I might hold---or need to hold.

David Parker said...

How about the uniformity of nature.

By the way logic is not self-evident in any sense I can determine.

Modus ponens is:

if p, then q
p
therefore, q

Now how do you "prove" modus ponens without presupposing it? You can't, which is why I think everyone at least presupposes the laws of logic and the uniformity of nature.

Adrian said...

Now how do you "prove" modus ponens without presupposing it?

Have you attended any intro logic courses? You can start by using truth tables, you can assume the negative and derive a contradiction, you can work it out from first principles. There is extremely little which must be presupposed and this includes logic (especially clear if you study multiple logical systems).

David B. Ellis said...


How about the uniformity of nature.


Already discussed. And no, I don't presuppose that. We might find that there are regions of the universe operating according to different laws. There's nothing intrinsically unreasonable about that possibility. And we can cross that bridge when we come to it (if we ever do). I don't have to hold presuppositions in that regard.

As to modus ponens, its self-evident to me that if the proposition "if p is true, q is true" is correct, and the proposition "p is true" is correct then the proposition " q is true" is also correct.

Things don't get much more self-evident than that.

David Parker said...

Tyro,

So now you're presupposing the truth functional operations of math?

David Parker said...

David Ellis,

You have violated the concept of a law. If things operate different elsewhere then it isn't a law.

Apparently we have different definitions of self-evident.

Scarecrow said...

After reading through all the "presupposing" bickering one thing seems to be clear. It's the confusion or comingling of the meanings of Axiom, First Principle, and Presuppostion.

They are not intechangable terms. But are being treated as such. Each has a specific meaning in their realms of use.

Axiom are used in mathamatics and logic and are not the same thing as presupposing. They are irreducable and or self evident starting points for which other principles may be logicly derived.

Presupposion within the christian belif system is a starting point for a worldview. Evidence and arguments are only marshalled after the fact in an attempt to justify the theological assumptions already made

IMHO science has as it's "First Principle" that the universe (all that exists) is a natural phenomena and can be explained by naturalistic means.

Food for thought gentlemen, carry on.

Adrian said...

David,

So now you're presupposing the truth functional operations of math?

No.

Logical systems and math can be built upon a very small number of axioms (far smaller than you seem aware of). Axioms, not presuppositions.

Now, are you trying to bring this back to the topic or is your aim to sling mud and stir up arguments?

David Parker said...

Tyro,

You barged in when I asked David Ellis a simple question. My aim was just to ask a simple question. Feel free to stop trying to look like mister-know-it-all.

Your distinction between axioms and presuppositions is fine with me, in that case then the law of non-contradiction is an axiom and not a presupposition. Semantics, semantics...

David B. Ellis said...


IMHO science has as it's "First Principle" that the universe (all that exists) is a natural phenomena and can be explained by naturalistic means.


If the supernatural existed and produced observable effects, the methods of science would be as capable of dealing with it as any naturalistic observable phenomena. If, for example, radiological and other data confirmed that the earth and, indeed, the cosmos was 6000 years old and other empirical evidence supported, over and over in every detectable way, biblical creationism we'd still be doing science.

Science does not presuppose naturalism. That you think it does merely demonstrates that you're as biased in your thinking as presuppositionalist apologists accuse you of being.

Forgive me if I'm not interested in joining in that.

David Parker said...

David Ellis,

Unless you think the universe is eternal, then you must think consider that as observable evidence of some cause that is not the universe, right?

District Supt. Harvey Burnett said...

David B Ellis,

PLEASE…you seethe with all kinds of presuppositional biases as Eric has adequately, correctly and thoroughly pointed out. NONE of you can honestly say that you follow the evidence where it leads. Your man HUME doesn’t say that and NONE of you radical metaphysical naturalists do either…the PROOF is in your comments and statements of denial which is certainly NOT a river in Egypt.

Russ,

I couldn’t pass this one up, first you call me a charlatan and submit a TOTAL lie like this regarding some “government studies”:

Russ: “Harvey, that these are atheist or liberal studies, let me tell you they are not. Most are simple rates determined from government gathered data on health and human welfare.”

Man don’t even talk to me...when is the US gov’t surveying people regarding religion and effects of religion on people. Maybe an independent study who offers a certain segment of data but you’re off in left field...get outta here!

Then you said this: “But, as with most Christian clergy, you cannot follow the morally correct path of counseling your congregation to invest their money in things that would brighten the future prospects for themselves and their families like education, healthcare, and physical fitness.”

Man you don’t even KNOW me. I am a FINANCIAL and INSURANCE ADVISOR, and COMMUNITY ACTIVIST who makes a living doing seminars in which I train and educate CHRISTIANS in matters of economics, simple and advanced wealth transfer, business establishment and tax minimization techniques. Then in my spare time I enhance schools through DAILY volunteerism, student counciling and administration support. How STUPID can you be????

It’s amazing atheists KNOWS so much as it pertains to the nonexistence of God but like David B Ellis says when asked about his own presuppositions regarding the uniformity of nature:

“I don't presuppose that. We might find that there are regions of the universe operating according to different laws.”

So get this, he presupposes that God does not exist because he can’t scientifically measure him, while at the same time he allows that “somewhere over the rainbow” laws may operate contrary to what science says that they do on Earth and in the known universe...Go figure...and they call this logic????

I'LL END ON A GOOD NOTE, Good lesson Carbonbased, I’ll make note of it and keep it for future reference.

Peace.

District Supt. Harvey Burnett said...

dANIEL ~ "There is no evidence that the historical person named Jesus that is described in the bible existed at all. Although it is reasonable to assume that there were many hundreds, if not thousands, of people named Jesus wandering around the ancient world."

There is AMPLE evidence from your statement that many people have lost their minds though...Even John Loftus doesn't hold to this atheistic fantasy...why?...EVIDENCE...We can gladly debate the details of Christianity, but you and those that hold this view don't have a clue in the middle of Blues Clues!!!

Later.

Scarecrow said...

"Science does not presuppose naturalism. That you think it does merely demonstrates that you're as biased in your thinking as presuppositionalist apologists accuse you of being."

Are you sure you know what form of naturalism I'm talking about? I don't think you do, there is metaphysical naturalism and methodological naturalism. I'm talking about the later. Science most certainly does use methodlogical naturalism as a starting point.

"If the supernatural existed and produced observable effects..."

Then David it would not be "supernatural" I'm sure at some point in time the idea of a invisable force like magnatism seemed supernatural, but if it has a naturalistic explination then it no longer IS supernatural.

A first principle is not the same thing as presuppostion. The supernatural being god is not self evident nor irreducable. It raises more questions than it answers. Nor is it an axiom.

David Parker said...

Carbon,

Apparently everyone wants to argue David Ellis' points for him.

I'm confused that you think anything with observable effects isn't supernatural. Am I hearing you correctly?

If so, then please provide me with a naturalistic explanation for the natural universe (hint: you can't).

Russ said...

So, Harvey,
I know that clergy can't discern reality from wishful thinking, but I'm going from your own information about your current and expanding ministries. You said to me:


Man you don’t even KNOW me. I am a FINANCIAL and INSURANCE ADVISOR, and COMMUNITY ACTIVIST who makes a living doing seminars in which I train and educate CHRISTIANS in matters of economics, simple and advanced wealth transfer, business establishment and tax minimization techniques. Then in my spare time I enhance schools through DAILY volunteerism, student counciling and administration support. How STUPID can you be????


but, in your blog profile, it says nothing about financial advisor while highlighting that,


About Me

Harvey Burnett has been a gospel minister for 26 years. He is celebrating 27 years of salvation in 2008. He is the husband of ONE wife of 23 years and the father of two children ages 6 and 11. Pastor Burnett is the founder of New Bethel COGIC of Peoria, IL. and the upcoming Bethel Epiphany COGIC of Detroit, MI.


So, what are people to believe, Harvey. Are you clergy as you, no doubt rather proudly, state in your profile, or did you contrive that to impress someone else? Can you at least be honest in this? Wow, you even call yourself "Pastor Burnett," but you deny it when it suits your purposes! Did you write your profile titled, "About Me"?

You appear to be quite the opportunist, Harvey, and now I'm sure that charlatan was not a strong enough word.

David B. Ellis said...


I don't think you do, there is metaphysical naturalism and methodological naturalism. I'm talking about the later. Science most certainly does use methodlogical naturalism as a starting point.


I know the difference and of course, in science we look for and expect naturalistic explanations since we've never found anything else and have no reason to expect to.

But calling it science's "first principle" is elevating a reasonable expectation (which is all methodological naturalism should be) to what sounds suspiciously like a dogma.

Perhaps you didn't mean it like that but that's the way it came across.


"If the supernatural existed and produced observable effects..."

Then David it would not be "supernatural" I'm sure at some point in time the idea of a invisable force like magnatism seemed supernatural, but if it has a naturalistic explination then it no longer IS supernatural.


I am NOT referring to things with an unknown but naturalistic cause (one with a physical basis). I'm referring to actually supernatural/magical things which, if they existed, produced observable effects.

The fact that something is observable does NOT make it naturalistic. If the wizards and witches and magical creatures of the Harry Potter novels existed we could observe them and perform scientific tests on them and do all the things science does with these actually supernatural phenomena just as we do with natural phenomena.

To say that something which produced observable effects is, by definition, naturalistic is absurd. Haven't you ever read a supernatural thriller or a fantasy novel? Are the werewolves, vampires, elves and other supernatural entities in the stories unobservable?

Of course not! The word "supernatural" does not mean "unobservable".

We just happen to live in a world where the supernatural doesn't actually exist---and so it has not been observed. That doesn't make it, in principle, unobservable. Nor the observable, in principle, naturalistic.

Honestly, I'm getting sick of having to explain this over and over to other nonbelievers. Why is this misconception so common when only the most minimal reflection reveals its utter absurdity?

David Parker said...

David Ellis,

Something exists. Now as opposed to dodging my question, let me lay out the options let me know which you subscribe to:

a) The universe always existed
b) The universe began to exist

It is not my position that all things observable are supernatural, I thought it was yours. What a relief to know neither of us think that! :)

If the supernatural existed and produced observable effects, the methods of science would be as capable of dealing with it as any naturalistic observable phenomena.

You are assuming that the material world itself is not an observable effect of divine causation are you not?

Hey Carbon,

I think I accidentally intercepted a comment you intended for David Ellis. Sorry about that the double David thing has me all confused! :)

David B. Ellis said...


Unless you think the universe is eternal, then you must think consider that as observable evidence of some cause that is not the universe, right?



I do not know what came before the big bang. At this point no one does. But that a scientific question is currently unanswered is not reason to expect the answer to be supernatural.

If you are presenting your question as a precursor to a first cause argument for theism you will be disappointed. There are naturalistic versions of a first cause (vacuum fluctuation models of the big bang, for example, to name only one).

David Parker said...

David Ellis,

Its simple, something doesn't come from nothing in a naturalistic universe.

That singularity either always existed or something else came before it, which means it really isn't singular.

David B. Ellis said...


You are assuming that the material world itself is not an observable effect of divine causation are you not?


No. I'm just not assuming that it is.

If you wish to present some variation of the first cause argument for theism then please stopping beating around the bush. Just present your argument and we can then discuss its merits.

David B. Ellis said...

You should keep up with your big bang cosmology if you want to discuss this topic. There are a wide range of models currently proposed and hotly disputed in the scientific community. Some do not even include a singularity as in the standard big bang model.

And I'm not interested in playing 20 questions. If you have a first cause argument to present then do so.

David Parker said...

David Ellis,

I'm only interested in examining what you've already said. You don't know what came before the big bang, but you also said you don't believe the supernatural existed.

So what I conclude from that is you think some natural entity preceded the big bang.

The basic kalam argument is this:

P1 Everything that begins to exist has a cause
P2 The universe began to exist
C Therefore, the universe had a cause

That's pretty basic, but if you want to give any thoughts on that variation I'm all ears.

David B. Ellis said...


That's pretty basic, but if you want to give any thoughts on that variation I'm all ears.




So far we have a proposition we both think likely to be true:

Something caused the big bang.

Would you now like to make a case for this cause being God?

Adrian said...

P1 Everything that begins to exist has a cause

This one is directly contradicted by observations and quantum field theory. It would be more accurate to invert that and say that as far as we know, everything which begins to exist does not have a cause. I don't want to go that far as I'd be making the same unevidenced leap of faith but it gives a clear signal that we must have proof for a cause before we assign one.

But lets face it, anyone interested in discovering anything about our universe uses physical theories with precise descriptions and predictions and evidence, not cutesy sylogisms. Just stating things and believing them unquestioningly is something we leave to the theologians.

Anonymous said...

David, since in my opinion you're being very evasive, let me take only what you've actually claimed -- what you've incontrovertibly claimed -- and work from there to justify my initial claim about the inevitability of metaphysical presuppositions.

First, let me clear up a misconception you seem to have: to say that you presuppose that P is not to say that you presuppose that P always has been and will be the case. In other words, presuppositions are defeasible. Therefore, you cannot escape the claim that you are relying on presuppositions by suggesting that you're open to changing your mind about them, or by claiming that you're not (logically) necessarily committed to them, or by claiming that you only accept that P because you have no reason to reject it at the moment.

Here is a list of some of your claims:

1. "I have no particular problem with taking a few thinks as "givens". Things that are simply self-evident. Mostly here I'm referring to logical truths."


2. "I don't agree, though, that we are obligated to have metaphysical presuppositions---to make assumptions about the nature of reality."


3. "I think what most of us naturalists (and what good science does) is observe the world and ask themselves, 'what model fits best with these observations'."


4. "Sure it does [i.e. science supports naturalism] ---it just doesn't prove naturalism (which would be impossible). In all the centuries we've been doing science all the things its investigated and which were thought to be supernatural have turned out to have naturalistic explanations (at least all the things we've so far discovered the explanation for)...And, no, this isn't disproof of the supernatural (there is always the bare possibility that we will find good verifiable evidence for it in the future). But it IS good reason to refrain from believing in the supernatural (which is all that my own naturalism amounts to---the absence of belief in supernatural phenomena until such time as good reason to be convinced is found)."



5. "If the ultimate "stuff" of which reality is composed behaves exactly the same way it doesn't make an iota of difference if you call it "idea" or "matter"---there is no substantive difference.
In such a case the materialism/idealism debate is just semantics."

6. I don't presuppose there is a world that exists independently of the observer; that the world is orderly and can be understood by human cognition; that matter has not just a chronological primacy over consciousness, but an ontological primacy over consciousness; that our senses and cognition can accurately perceive and conceive reality; that our memories are reliable.

7. "I observe that the world of my sensory experience is orderly and manage to make predictions within it which would not be possible if there was no order to be found."

8. "I do not think our senses are accurate in their perception of reality. We often, in fact, misperceive. I don't assume that our senses contact "reality" at all, for that matter."


9. "Our memory appears to be terribly fallible. Of course, if both my memory and my perception is in total error then I can never know anything at all. I have no particular reason, though, to think they are. I deal with the sensory perceptions and memories I have. This does not involve an assumption of their reliability (in fact, I think they are likely to be in error fairly often)---only a recognition that there is no workable alternative to doing so."

10. "If the supernatural does not exist, science cannot do anything but never confirm the existence of the supernatural (exactly what has so far taken place). There are all manner of observations science could have made though (I listed many) which would have conclusively disconfirmed naturalism and confirmed the supernatural."


11. ("What makes the fundamental "stuff" of reality if it is called idea different from one where its called matter if there are no difference in the way they work.")


12. "So obviously I do think science can provide support for metaphysical theories."


13. "My naturalism consists of the absence of belief in supernatural phenomena---and I am defining supernatural phenomena to be phenomena where mind acts, exists, gathers knowledge, etc without any physical basis for doing so...
In that sense it can be described as a variety of physicalism---and, again, consists simply in the absence of belief in phenomena with no physical basis.
If provided with evidence of such phenomena that absence of belief would be gone."

14. "It [science] provides support for the proposition 'there is no basis for belief in the supernatural'."



15. "The sensory world is the only world we have available to us---so its the one I deal with. I think the hypothesis that it reflects an actual external world is the most plausible but don't assume I'm right about this being the case. I'm willing to entertain "brain in a vat" scenarios. They are, after all, unfalsifiable. Therefore, it cannot be said that I hold the presupposition that there is an external world at least somewhat accurately reflected by my senses. At best, I consider it the most sensible working hypothesis (mostly on the grounds of occam's razor)."

16. "I observe orderly predictable phenomena and act accordingly."


17. "So there's no basis for not employing induction (acting on the basis of past observations of the way the world is ordered)."


18. "My only metaphysical opinion so far discovered is a negative variety of naturalism (which consists of an absence of belief in the supernatural). There doesn't seem to be much presuppositional potential in that."


19. "My memories include examples of my memory being shown to be mistaken (if only it didn't---I'd have gotten straight 100's in history). So, at best, my memory is only partially reliable...if my memory is always wrong then I'm screwed anyway. My only option is to use the memories I have in thinking about the world."

20. "In strict philosophical terms I'm a pretty thoroughgoing skeptic. You're going to have a tough time trying to making me out to be a presuppositionalist in any substantial sense."

David, before I continue, I have a question: Can you *honestly* say that you don't see any metaphysical presuppositions hiding among these twenty claims?

David Parker said...

David Ellis,

Do you agree that the cause must be non-material?

David Parker said...

Tyro,

P1 Everything that begins to exist has a cause

This one is directly contradicted by observations and quantum field theory.

What observations have you made that would disconfirm P1?

Exactly how does quantum field theory falsify P1?


It would be more accurate to invert that and say that as far as we know, everything which begins to exist does not have a cause. I don't want to go that far as I'd be making the same unevidenced leap of faith but it gives a clear signal that we must have proof for a cause before we assign one.

There is a chair in my room. What proof must be garnished for causation to be accepted?

David B. Ellis said...


David, before I continue, I have a question: Can you *honestly* say that you don't see any metaphysical presuppositions hiding among these twenty claims?


There might be. Feel free to point one out if you can find one.

I think it needs to be said, though, that I'm operating according to a different epistemological approach from that of presuppositionalism. The presuppositionalist claims that we start with metaphysical (and other) presuppositions and build up our worldview from there.

I think it makes more sense to make ones starting point the body of observations and experiences available---not preheld metaphysical assumptions--- and to compare them to various models of the world to see what seems to best account for them.

There's not much room in that approach for presuppositionalism. Try as you might I don't think you're going to be able to find metaphysical presuppositions in my worldview. That's may be how YOU do metaphysics but my way is pretty much the polar opposite.

David B. Ellis said...

But since you seem to embrace the idea of metaphysical presupposition more than I do please tell us what you think what it is that makes a particular presupposition reasonable and another not reasonable.

Or do you consider all presuppositions to be created equal?

:)

What presuppositions do you think one should hold and why?

mg01 said...

First of all just as the atheist begins with the presuppositional premise that God does not exist, the Christian by EXPERIENCE begins with the presuppositional premise that God DOES exist.

Is the presuppositional premise that Brahma exists necessary when examining the claims of Hindu? Or Allah for Islam?

All have to be approached equally. Failing to do so is to employ a double standard and exhibits bias toward a predetermined conclusion.

Adrian said...

David,

What observations have you made that would disconfirm P1?

Create a list of all things which we know "began to exist" and which we can observe. A short list includes so-called Virtual Particles which were predicted by quantum field theory to spontaneously spring into existence. Using QFT, physicists predicted what influence VPs would have and have confirmed their existence using spectroscopy and the Cassimir Effect.

The same basic mechanism is thought to be the explanation for how all of the matter in the universe arose. You're welcome to propose an alternate mechanism but it will have to be stronger than current quantum field theory, a daunting challenge. At the very least, simply asserting that QFT is wrong doesn't cut it.

You've made a strong positive statement that is in direct contradiction to modern physical theories. You need to back up this claim with some evidence and not just a "well it makes sense" hand-waving. One thing we've learned over the past 100 years is that what makes sense in our macroscopic, Earth-bound intuition is no guide at all to what's actually happening in the quantum level.

Anonymous said...

Hi David

You wrote: "I think it needs to be said, though, that I'm operating according to a different epistemological approach from that of presuppositionalism. **The presuppositionalist claims that we start with metaphysical (and other) presuppositions and build up our worldview from there.**
I think **it makes more sense to make ones starting point the body of observations and experiences available---not preheld metaphysical assumptions--- and to compare them to various models of the world to see what seems to best account for them.**"

That certainly isn't at all what I mean when I speak about presuppositions. Let's take an example:

Bill: "What do you think of my new car?"
Pete: "It's great!"

We hear conversations like this all the time. Now, does Pete, when he's asked the question above, *begin* by thinking, "My friend Bill exists; I exist; this car exists; This car isn't one I've seen Bill with in the past; there's a legal relationship between people and objects called 'ownership'; the relationship between Bill and this new car is one of 'ownership'; it's possible to pass judgment on this new car on a variety of grounds: aesthetic, economic, functional, etc.; When Bill and I use words, we mean roughly the same things, so I can make sense of his question, and he can make sense of my answer; and so on. Obviously, all these notions (and a host of others) are presupposed by Pete in his answer ("It's great!"), and these presuppositions themselves have presuppositions. (Of course, Pete may, for example, think that Bill is lying, and that the car isn't his, but this still presupposes ownership as a relation, the car's existence, etc. and adds a new set of presuppositions about Bill's tendencies, about lying in general, etc.)

Now, a proposition is what declarative sentences express, and thus must be either true or false (whether we know the truth value); a presupposition is an implicit assumption that's a necessary condition for a statement being either true or false (Strawson). As an assumption, it's related to the proposition with respect to the background beliefs the proposition takes for granted. From these two basic definitions (which, admitedly, skate over a lot of complications, e.g. multi-valued logic, problems of implicature, etc.; however, they will do for now) we can see that we *don't* in any way 'start' with presuppositions, as you suggest; rather, they are inevitably built into the structure of whatever it is we're saying. Every proposition contains presuppositions that are necessarily similar in kind to the initial proposition; you can't avoid them. Therefore, it follows inescapably that metaphysical propositions (i.e. propositions with metaphysical content, not propositions about metaphysics) must contain metaphysical presuppositions.

Now, if we keep this in mind, as well as my earlier point (in my last post) that presuppositions aren't indefeasible, we can look at one (for the sake of brevity!) of your propositions and try to uncover some metaphysical presuppositions (understood as metaphysical propositions implicitly assumed by one of your explicit propositions, that contain a necessary condition for the truth or falsity of the initial proposition).

3. "I think what most of us naturalists (and what good science does) is observe the world and ask themselves, 'what model fits best with these observations'."

Can you both assert this proposition and deny that 'the world' and 'the observer' both exist, that they exist in some relationship to each other, that they therefore cannot be identified, that the world is such that it provides the observer with information capable of modelling, that the model cannot be identified with the world, etc.? Absolutely not; if you did, you'd be guilty of a number of contradictions. For example,

"I think what most of us naturalists (and what good science does) is observe the world and ask themselves, 'what model fits best with these observations,' and neither the world nor the observer exist."

"I think what most of us naturalists (and what good science does) is observe the world and ask themselves, 'what model fits best with these observations,' and there is no relationship between the world and the observer."

"I think what most of us naturalists (and what good science does) is observe the world and ask themselves, 'what model fits best with these observations,' and we can identify the world with the observer."

"I think what most of us naturalists (and what good science does) is observe the world and ask themselves, 'what model fits best with these observations,' and the world cannot be modelled."

"I think what most of us naturalists (and what good science does) is observe the world and ask themselves, 'what model fits best with these observations,' and our models and the world are indiscernible."

Here, we have five obvious contradictions: you can't claim that naturalists observe the world, and that neither the world nor the naturalist exist; you cannot claim that naturalists observe the world, and that there is no relationship between the naturalist and the world; you cannot claim that naturalists observe the world, and the world and the naturalist are indiscernible; you can't claim that the naturalist models the world, and that the world can't be modelled; and you can't claim that the naturalist models the world, and that the naturalist and the world are indiscernible.

From here, we can draw out a number of metaphysical presuppositions:

1. The world exists.
2. The world is of such a nature that it can be observed (whether in some strong sense, or in a minimal sense).
3. At least one observer exists.
4. The observer stands in a relationship to the world such that it's possible to observe the world.
5. We can distinguish the world from the observer, not in the sense that the observer isn't part of the world, but in the sense that the observer stands in a relationship to the world.
6. That the relationship between the observer and the world is such that the observer can develop models of the world.
7. The world is of a nature that makes modelling it seem possible.
8. The world is of such a nature that some models 'fit' it better than others.

And so on. Of course, we could also proceed to draw out the metaphysical presuppositions of your presuppositions, but I think I've done enough so far to establish my point.

Anonymous said...

Tyro, from what I understand, virtual particles don't violate causation for two reasons: first, they don't transmit information; and second, causality isn't reducible to sufficient conditions, but can be understood in terms of necessary conditions as well (e.g., here the presence of a quantum field is a necessary condition).

Note, I'm no quantum physicist (but then, I doubt any of us here are), so I'm relying on what I've read by quantum physicists.

David B. Ellis said...

I think an early question was raised and hasn't been sufficiently responded to:

What reason do we have to not believe in vampires, fairies, werewolves, and other supernatural entities that aren't commonly believed in in our own culture?

Or, for that matter, gods not commonly believed in by us. Zeus, Thor, Osiris, etc.

I think this issue is more illuminating than its so far been given credit for being.

Adrian said...

Eric,

Tyro, from what I understand, virtual particles don't violate causation for two reasons: first, they don't transmit information; and second, causality isn't reducible to sufficient conditions, but can be understood in terms of necessary conditions as well (e.g., here the presence of a quantum field is a necessary condition).

The question of "causality" grows difficult at quantum levels and so discussions about violations are often translated to questions about violating "locality". VPs do not violate locality.

The second point about preconditions is fuzzier. When a frayed wire in on a toaster catches fire and burns down the house do we say that the cause was the presence of oxygen in the house? Certainly it was necessary.

Talking about the presence (or more accurately the absence) of fields as "causing" VPs is even worse. There was no agent, no actor, no trigger, no impetus, nothing which we would describe as a cause. They arise due to the nature of quantum uncertainty in fields - this really stretches causality. If Christian apologists are willing to water down the subject of causality this far, they have reduced the "cause" of the Big Bang to this same level, a "cause" no more significant than the void between fields, not even an agent or an entity but a void or a line in a physical theory.

I think it does harm to our ability to communicate when apologists redefine words like this whenever they hit problems. If your statement hits a problem an honest person should say "Everything that we know which began to exist did so due to physical laws operating in our universe." This is accurate but clearly it doesn't get the apologist anywhere so they keep with the original formulation and then try a bait-and-switch by redefining "cause" when pressed, yet switching it back to be a personal actor, a cause like a boy causing a window to break by throwing a baseball when we've seen it's nothing of the sort.

Scarecrow said...

"From what I understand, virtual particles don't violate causation for two reasons: first, they don't transmit information;"

I'm not sure what you mean by "transmitting information" but.
Virtual particle coming into existence and annihilating one another does cause a measurable force. I believe this would be "information."

David B. Ellis said...

Eric, I think your usage of presupposition will require further clarification before I can discuss it. You seem to be using it in a different sense from that of the presuppositionalist christian theologians I've read.

So, to keep it focused on the topic of the existence of God, what do you understand the following phrase to mean:

Cornelius Van Til presupposes the existence of God.

or, to it slightly differently:

Van Til holds as a presupposition of his worldview that God exists.

Anonymous said...

Tyro, it is actually physicalism that sets up necessary conditions (in addition to sufficient condtions, etc.) as causes. Take any mundane causal series -- say, the one that begins with light from the sun, and ends with your seeing a tree -- and look at it for a moment. Now, given physicalism, why should we say the light from the sun is the cause of my seeing the tree, and not the fusion that results in the release of energy that results in the light? Or why not the laws(regularities) that govern fusion? Why not the events that led to the formation of the sun, etc.? There are no non-arbitrary grounds, given physicalism, for choosing one set of conditions over another, since what nature gives us is an unbroken sequence (whether determinate or indeterminate); rather, it is only when we take into account our mind-dependent interests (goals, or final causes) in understanding causal sequences that we can come up with non-arbitrary criteria for isolating causal sequences and claiming, "X caused Y." So, I don't think it's the case that the apologist is 'changing definitions' to suit himself, but merely that he is 1) availing himself of the physicalist's assumptions -- a perfectly respectable strategy; and 2) demonstrating that the physicalist's conception of causality is in serious disrepair!

Anonymous said...

David, I think I can clarify it much more easily by saying that I'm not a presuppositionalist (in the Van Til sense), and that I'm using the term 'presupposition' in the sense that philosophers -- secular or theistic -- use the term (which is not very different from the way the term is used in linguistics).

David B. Ellis said...

To be clear, when I say I don't have metaphysical presuppositions I'm using that term in the sense its used by christian presuppositionalist theologians and apologists like Van Til.

That may not be the same usage of the term you employ---but if you're criticizing my claim then you have to do so in terms of the definitions I was employing---not some other one you substitute---else you're just criticizing a strawman version of my position.

David B. Ellis said...

Remember that the context of this discussion of presuppositions was in response to Harvey's comment:


First of all just as the atheist begins with the presuppositional premise that God does not exist, the Christian by EXPERIENCE begins with the presuppositional premise that God DOES exist.


Which seems to have clearly been said out of the Van Tilian variety of presuppositionalism. And that's what I've been responding to.

Anonymous said...

David, I think we have to make a few distinctions here. First, there is the definition of the term 'presuppositon'; second, there is the claim that all propositions contain presuppositions; third, there is the claim that all propositions contain certain *very specific*, *necessary* presuppositons.

Presuppositional apologists make use of all three elements, but I'm only using the first two. I should add that you never made it explicit that you claimed not to hold 'specific' metaphysical presuppositions (which is all a denial of presuppositionalism requires); rather, you claimed not to hold any! This clearly justified my interpretation of your use of the term, and my restriction of it to the first two elements I mentioned above. In that sense, I don't think it's fair to accuse me of setting up a straw man.

Anonymous said...

David, actually the whole discussion began with my comments on this particular remark of Jeffery's:

"I get so tired of hearing about "presuppositions." That concept does not exist in the world of science, because the moment you introduce such a concept is the moment you're leaving science and entering the world of the imagination."

This is clearly not limited to the presuppositionalist's understanding of the term.

Anonymous said...

Let me clarify: I meant the whole discussion *between us* began with my remarks on Jeffrey's comments...

Adrian said...

Eric,

Take any mundane causal series -- say, the one that begins with light from the sun, and ends with your seeing a tree -- and look at it for a moment.

I think you need to come up with a better analogy. In this case you're considering a set of events, each event provides an impetus or trigger which leads directly ("causally") towards the next. We can start with two atoms as actors, fusing to emit a proton which strikes an atom in the tree, causing an electron to jump to a higher energy state before falling and emitting its own electron which... And so on. In each step there are specific particles or agents which interact with other agents.

Nothing like that is present with VPs. There are no agents, there are no triggers, there are no interactions. You cannot trace their existence back to any event at all.

I would agree with you that we could say that oxygen in the house "caused" a fire (it would contort our common-sense definition, but not it wouldn't stretch it too far). But could we say that the chemistry/physics of how atoms form bonds "caused" the fire? Without chemistry there could be no fire, but chemistry doesn't act it just describes.

That's the state we have with VPs. The quantum fields aren't interacting with anything to cause VPs, they arise spontaneously as a result of the nature of the physics in our universe.

Again, if this is the sense in which things which "begin to exist" are "caused", then the God we're left with which "causes" the BB is just the laws of physics. It requires a redefinition of "cause" and of "god".

a helmet said...

When I review all these statements about presuppositions, let me say that human reasoning, especially philosophy is utterly unable to find a firm contingency ground, fully unable to find a ground of absolute knowledge as a basis for building further theories -- presuppositions.

It isn't possible for sciences to "know" on an absolte basis, so presuppositions are never based on a solid ground.

You are rather in the position of blind men who poke in the dark with their white canes trying to see some truth where there is nothing to see. All these discussions seem like controversies among the blind about colors, and every blind man trying to guide the other blind ones through the darkness. Aren't they all going to fall into a pit? Most assuredly.

So philosophical speculations never find a contingency ground, no absolute knowledge as a basis.

So in fact this reminds me of someone like Münchhausen, who was plunged in the swamp and claimed to have dragged himself out of the swamp by his hair. (Together with his horse, of course!) Well, we know that such doesn't work, does it? If you are in the swamp there is no ground. There is nothing to firmly stand on. So what chance do you have when trapped in the swamp? You can only be saved out of it by an external force. This i why, according to Christianity, humanity can only be saved and not save itself.

Really, in the darkness (=no light = no sight = no knowledge) absolutely nothing can be known. This finally only leads to Socrates wisdom who had the insight "I know that I know nothing" which is probably the wisest conclusion any natural man can get in this earthly journey! And note, even this is not a sure knowledge, for those in the darkness do not even "see" that they are in the darkness. In the darkness absolutely nothing can be seen (=known). You only stray around with your white cane and poke around in the unknown.

Suppose you are located in a realm where there is no firm ground, then you'll never find any.

So you'd better be in accordance with Socrate and admit I know that I don't know anything.
That's the corner from which one can flee to the light. Just as no one can save himself out of the swamp, so no one can know anything in the darkness, but only escape by faith in the light . In your reasoning, Socrate's presupposition is the only presupposition I can think of that makes sense.

Anonymous said...

"I think you need to come up with a better analogy. In this case you're considering a set of events, each event provides an impetus or trigger which leads directly ("causally") towards the next...Nothing like that is present with VPs."

I wasn't using the 'sun/tree' example as an analogy (with respect to virtual particles), but merely as an example of the problems physicalists face when it comes to understanding causality. In this sense, it seems to me that your primary criticisms, which are premised on my use of it as an analogy, are beside the point.

"Again, if this is the sense in which things which "begin to exist" are "caused", then the God we're left with which "causes" the BB is just the laws of physics. It requires a redefinition of "cause" and of "god"."

Remember, I was referring to causation given physicalism; Christians aren't physicalists, so they don't face this problem. The Christian can use the logical consequences of physicalism to demonstrate the problems it faces with respect to causality without committing himself to that conception of causality.

David B. Ellis said...


David, actually the whole discussion began with my comments on this particular remark of Jeffery's:


Harvey's comment was prior to that. And it's entirely in the context of christian presuppositionalist epistemology and apologetics that I've been making my comments.

I am, of course, aware of other usages of the word "presuppose". For example, when I say that my wife told the truth when she denied putting the dent in my car this proposition presupposes (has as a necessary condition) that I have a wife---else the proposition makes no sense.

But that's not the usage I've ever been using in this discussion---nor any other but that of christian presuppositionalism and reformed apologetics (basically the religious epistemology of Cornelious Van Til and those influence by him).

David B. Ellis said...


fully unable to find a ground of absolute knowledge as a basis for building further theories -- presuppositions.

It isn't possible for sciences to "know" on an absolte basis, so presuppositions are never based on a solid ground.


Fortunately presuppositionalist epistemological systems are not the only options available to us.

Of course, no epistemology provides certainty.....if that's what you're after you're just chasing phantoms.

Anonymous said...

"Of course, no epistemology provides certainty"

Excepting the epistemic grounds for this proposition, eh? ;)

District Supt. Harvey Burnett said...

Russ,

You're a NUT...the ONLY thing I hate worse than an Ivory Tower religious person is an Ivory Tower atheist...

I AM ALL the above Russ...I have MULTIPLE occupations. I RECEIVE NO INCOME from the churches that I Pastor. I put my MONEY, TIME, ENERGY and RESOURCES where my mouth is...and in my SPARE TIME I come here to read and debate opinions of what I believe to be some good people (YOU OBVIOUSLY EXCLUDED)...

If this is proof of how an atheist thinks (And if I were an atheist I'd disavow your representation in any fashion)...then it is also proof of how they never give the benefit of the doubt but always questioning BEYONED reason...

Maybe RUSS you can tell me more about MYSELF, than I about a city of 130,000 or so can personally witness and have witnessed...What is it, you couldn't click the link on my blog regarding Health Insurance (Bottom link Left side) are you TRUTH challenged? Just unable to understand that Christians such as myself do more good in ONE MINUTE of their existence than people like you do in a lifetime???

I don't know...or maybe yet go to my local news paper at www.PJStar.com and put my name (Pastor Harvey Burnett)in the google search and see what comes up...

So I guess in your world a person such as myself can't do multiple things...I guess this is based on your own presuppositional thinking...

By the way the next time we sponsor a back to school program as I have also done for many years (I have the TV interview if you want proof) let me see if I can get you to match the $3,000 that I usually put in personally without taking a tax deduction for it...

Give me your email, since you have so much to say about charlatans and let's see who the REAL charlatan is on this board...Let's see if you can help people just for the sake of helping people with no promise of any return or personal benefit LIKE ME...

As you can see, I don't have too much to say to you in way of arguments and I'd prefer that you not address me outside of the arguments and topics here because you prove that you're an idiot.

Sorry everyone, please ovelook this occurance but for those who know me on this board, they also know I've handled this jerk rather nicely as compared to others who've presented themselves in a similar manner...

Thanks.

District Supt. Harvey Burnett said...

Eric ~ "we can see that we *don't* in any way 'start' with presuppositions, as you suggest; rather, they are inevitably built into the structure of whatever it is we're saying. Every proposition contains presuppositions that are necessarily similar in kind to the initial proposition; you can't avoid them. Therefore, it follows inescapably that metaphysical propositions (i.e. propositions with metaphysical content, not propositions about metaphysics) must contain metaphysical presuppositions."

YOU ARE DA MAN! YOU GO BOY!!!

Lovin' it.

Thanks.

David Parker said...

David: What observations have you made that would disconfirm P1?

Tyro: Create a list of all things which we know "began to exist" and which we can observe. A short list includes so-called Virtual Particles which were predicted by quantum field theory to spontaneously spring into existence.

So your claim is that anything we observe didn’t begin to exist?

Tryo: Using QFT, physicists predicted what influence VPs would have and have confirmed their existence using spectroscopy and the Cassimir Effect.

A lot of words, but little explanation for how exactly you propose Quantum theory defies causation.


The same basic mechanism is thought to be the explanation for how all of the matter in the universe arose. You're welcome to propose an alternate mechanism but it will have to be stronger than current quantum field theory, a daunting challenge. At the very least, simply asserting that QFT is wrong doesn't cut it.

Apparently you think you’ve presented an argument?


You've made a strong positive statement that is in direct contradiction to modern physical theories. You need to back up this claim with some evidence and not just a "well it makes sense" hand-waving. One thing we've learned over the past 100 years is that what makes sense in our macroscopic, Earth-bound intuition is no guide at all to what's actually happening in the quantum level.

I outlined an argument and once again you felt the need to defend David Ellis, who apparently has chosen not to respond.

Adrian said...

David,

So your claim is that anything we observe didn’t begin to exist?

No. As I think I said clearly, everything we observe which "began to exist" was not caused. If you read my responses to you and Eric, I think I've made that clear.

A lot of words, but little explanation for how exactly you propose Quantum theory defies causation.

I explained twice. If there's something specific you don't understand ask specific follow-up questions. What don't you understand, what is your current thinking, what have you done to educate yourself or answer your own questions?

I outlined an argument and once again you felt the need to defend David Ellis, who apparently has chosen not to respond.

You've made assertions which are contradicted by observations, hence your argument is invalid. Defend it or abandon it, simple restating it isn't an option if you have any concern for intellectual integrity or truth.


As for David Ellis, this is a public forum and if you're absolutely insistent on carrying on a private conversation, use e-mail otherwise accept that you'll get multiple responses and that people may come and go as their schedule permits.

David Parker said...

Tyro,

No. As I think I said clearly, everything we observe which "began to exist" was not caused. If you read my responses to you and Eric, I think I've made that clear.

I've seen assertions, but not argument. Can you defend this position on causation or not?

I explained twice. If there's something specific you don't understand ask specific follow-up questions. What don't you understand, what is your current thinking, what have you done to educate yourself or answer your own questions?

My question is simple, can you explain how Quantum Theory defies causation? Hint, using buzz words isn't an argument.


Defend it or abandon it, simple restating it isn't an option if you have any concern for intellectual integrity or truth.

Don't blame me if you can't defend your own position. I simply outlined an argument for Mr. Ellis and now you would have me believe things began to exist without cause, and to boot you feel you have secured your claim. I believe your intellectual integrity is in need of examination here Tyro.


As for David Ellis, this is a public forum and if you're absolutely insistent on carrying on a private conversation, use e-mail otherwise accept that you'll get multiple responses and that people may come and go as their schedule permits.

I'd just like to see Mr. Ellis' response, but apparently he has either departed the discussion or thinks you have taken over for him.

Adrian said...

david,

My question is simple, can you explain how Quantum Theory defies causation? Hint, using buzz words isn't an argument.

If you're talking about Quantum Field Theory not Quantum theory and if you're referring to how VPs appear uncaused and not some nebulous "defies causation", then yes, I can. I have done so. If you have problems, tell me what they are and I'll elaborate. I've already spent considerably more time defending this position than you've put into your unsupported, unevidenced syllogism.

I'm not going to try to guess what you do and don't understand. I'll extend you the courtesy of assuming that you would only make sweeping, universal statements of how matter behaves after researching the subject fully. So instead of trying to summarize 100 years of physics, why don't you just tell me what you have a problem with.

David Parker said...

Tyro,

I take that as a concession.


Recall again what I originally said to Mr. Ellis,

The basic kalam argument is this:

P1 Everything that begins to exist has a cause
P2 The universe began to exist
C Therefore, the universe had a cause

That's pretty basic, but if you want to give any thoughts on that variation I'm all ears.



Now Mr. Ellis agreed that the proposition was true, so I continued on to discuss the implications of the argument with him.

You, on the other hand, dove into a refutation of P1, and now seem to be frustrated that I'm not convinced by your hand-waving and buzz words. Instead you want me to do all the work and ask questions. Hey dude, you're the one that said this:

This one is directly contradicted by observations and quantum field theory. It would be more accurate to invert that and say that as far as we know, everything which begins to exist does not have a cause.

Now you're tossing onus around:

Defend it or abandon it, simple restating it isn't an option if you have any concern for intellectual integrity or truth.


I simply outlined a variation of the cosmological argument to discuss with Mr. Ellis, but you wanted to go for gold and refute P1. Hey that is great if you can do it man, but so far you haven't. Instead you appear to be having a meltdown or something:

If you have problems, tell me what they are and I'll elaborate. I've already spent considerably more time defending this position than you've put into your unsupported, unevidenced syllogism. I'm not going to try to guess what you do and don't understand. I'll extend you the courtesy of assuming that you would only make sweeping, universal statements of how matter behaves after researching the subject fully. So instead of trying to summarize 100 years of physics, why don't you just tell me what you have a problem with.


I'm not going to substantiate your claim for you, but I'll try to help you along by restating the question:

How exactly do Virtual Particles begin to exist without a cause? You mean that since no cause is detected, it doesn't exist right?

Adrian said...

David,

How exactly do Virtual Particles begin to exist without a cause? You mean that since no cause is detected, it doesn't exist right?

No, I don't mean that.

VPs are a direct prediction of Quantum Field Theory which states (roughly) that like the uncertainty in a particle's position and momentum, there is also uncertainty in the level of any field. When any field mediators are not present the level of the field would be zero which is prohibited and so QFT predicts that particles will wink into and out of existence. In particular this means they arise only when there are no other particles, actors, agents, triggers, events or anything else which might be called a cause.

They aren't objects which we knew existed and we just scratched our head asking "Gee, I wonder what caused them", they were a prediction of QFT and they were only observed as a result of this prediction. It isn't merely that we don't yet know a cause, everything we know about quantum field theory says that there is no cause.

Now for the Nth time, do you have anything to substantiate your original claim or not?

David Parker said...

Tyro,

Apparently you suffer from reading incomprehension.

J. K. Jones said...

Interesting. You assume from the outset that there is no positive evidence for God’s existence. I have provided some arguments on my blog, but I’d prefer to respond to your post. Here goes, quoting form the original post.

“…that God depends upon the revelation found within the pages of the canonized writings in the Bible…”

God’s existence has been substantiated by logical argument not dependent on Scripture.

“…intense undeserved suffering in the world which cannot be explained by a perfectly good omnipotent creator…”

So what is the alternative? For this to really hold up, you have to have an objective, logically established basis for having “undeserved suffering.” There has to be a rational standard for right and wrong for suffering to be undeserved. Otherwise you are borrowing form the Christian worldview to criticize the Christian worldview.

Someone without a reference for good and evil cannot have evil in their world at all. Any example of evil or undeserved suffering becomes an example of something truly evil. If no basis for good and evil is given, nothing is evil. These examples of true evil then establish a standard of good. They become arguments for Christianity instead of arguments against it.

Also, you are assuming “perfectly good omnipotent creator” does not have a good reason for suffering. He might have a good reason after all.

“…there is archaeological evidence against the Biblical stories of the world-wide flood, the Exodus and the conquest stories in the Bible…”

There is more archaeological evidence for the Bible’s accounts than against them.

“…there is geological evidence showing the earth has existed for 5 billion years…”

Not all Christians believe in a ‘young earth. What is your response to the Christians who do hold that the world is 5 billion years or so old?

“…there is biological evidence showing one species evolved into the next one…”

I’d like to see some direct evidence of this. Show me an example of an intermediate form between two species in the fossil record.

Besides, how could a random process of evolution result in a scientist that could argue for a scientific / logical / mathematical explanation for this process? How can random events result in a non-random intelligence? To even reason at all, we have to have a non-random reasoning process, an ability to determine information from sense perception, a world which is generally predictable (non-random), and a world that functioned in the past as it does in the present and will in the future. These things must be designed into the universe by a rational designer.

“…no death in the Garden of Eden…”

Maybe it was just death of a particular species, namely man.

“…we believe and behave as we do based upon early childhood experiences…”

I’d like to argue with you about this, but my early childhood experiences have formed me in such a way as to pre-dispose me for Christian theism.

“…neurological evidence in that strokes and seizes disconfirm the notion of a soul…”

First, you need to make up your mind if it is nature or nurture that determines us. These things are somewhat contradictory.

I’d also like to argue with you about this, but, in addition to the way my early childhood experiences have formed me, my genetic material pre-disposes me to believe in Christianity. Argument is now a series of entirely useless semantic exercises.

With the last two comments, you have established that there can be no reason to have a reasonable argument. We are all just pre-disposed to our opinions. Since you seem to want to put forth reasonable arguments, you give the lie to your own position. Your insistence on early formative and genetic processes forming our opinions makes it unreasonable to provide a rational argument against those opinions.

You really should just spend your time looking for a drug to counteract religious belief. That would be more consistent.

As John H. Gerstner used to say, “It stands to reason, that there can be no reasons against reason.”

“…historical evidence against the believability of the virgin birth story, Satan, hell and the bodily resurrection of Jesus from the dead too…”

What is the nature of this historical evidence? Christians have eyewitness testimony that affirms these very things. Evidence has been provided.

JK

Charlie said...

Tyro,

Please learn to read more carefully. I did offer a counterexample to your formulation of Dawkins's argument (so did Darrin, for that matter). An object could exist and be complex "in any way" yet lack proper physical parts. You have not identified any problems with this possibility. So we still have no reason to think your formulation of Dawkins's argument is sound.

About the theistic Kalam argument.
Virtual particles do not begin to exist from nothing without a cause. You're using an antiquated objection. Quentin Smith (chief critic of the Kalam) doesn't even use virtual particles as a counterexample to the causal premise of the Kalam anymore, as his position now, confirmed by science, is that the presence of a quantum mechanical vacuum would be a sufficient reason for the existence of virtual particles -- virtual particles are probabilistically caused by the vaccua.

Craig himself has also addressed this years ago. The vacuum which causes the virtual particles to exist is certainly not analogous to "nothingness", so the example isn't even relevant to ex nihilo nihil fit (out of nothing, nothing comes), which is at work in the causal premise.

As Barow and Tipler remark:

"...the modern picture of the quantum vacuum differs radically from the classical and everyday meaning of a vacuum-- nothing....The quantum vacuum (or vacuua, as there can exist many) states . . . are defined simply as local, or global, energy minima (V'(O)= O, V"(O)>O)" (Barrow & Tipler, The Cosmological Anthropic Principle, p. 440)

Here is where Craig addressed this (over a decade ago):

"The microstructure of the quantum vacuum is a sea of continually forming and dissolving particles which borrow energy from the vacuum for their brief existence. A quantum vacuum is thus far from nothing, and vacuum fluctuations do not constitute an exception to the principle that whatever begins to exist has a cause." (Craig, The Caused Beginning of the Universe: a Response to Quentin Smith." British Journal for the Philosophy of Science 44 (1993): 623-639.)

David Parker said...

Tryo,

First I agree with the distinction between necessary and sufficient conditions. Obviously a quantum vacuum is a physically necessary condition of a virtual particle's existence, therefore in that sense VP's have causes.


Sorry if I'm a bit short today, but I will be busy at work for awhile. Aside from the obvious dispute over whether VP's really exist, here are a few responses from Dr. Craig:


"the central point to be made here is that the quantum mechanical vacuum on which [virtual particles] depend for their existence is emphatically not nothing. The dynamical properties of vacuous space arise out of its interaction with matter and radiation fields, in the absence of which 'this dynamism of empty space is but a formal abstraction lacking physical reality.' The quantum vacuum is a sea of fluctuating energy which gives rise to virtual particles. Thus, virtual particles can hardly be said to arise without a cause."


"Wholly apart from the disputed question of whether virtual particles really exist at all, *24 the central point to be made here is that the quantum mechanical vacuum on which they depend for their existence is emphatically not nothing. The dynamical properties of vacuous space arise out of its interaction with matter and radiation fields, in the absence of which “this dynamism of empty space is but a formal abstraction lacking physical reality.” *25 The quantum vacuum is a sea of fluctuating energy which gives rise to virtual particles. Thus, virtual particles can hardly be said to arise without a cause

Oppy apparently thinks that cosmological models in which the universe originates via a spontaneous fluctuation from the primordial vacuum are distinct from models in which the universe does not violate the mass-energy conservation law because the sum total of its positive and negative energy is zero. But this is just confused: these are the same models, all presupposing the existence of the quantum mechanical vacuum which spawns the universe.*26 Thus, these models do not subvert the causal premiss. Moreover, while these models merited scientific discussion when Davies wrote God and the New Physics back in the early 1980’s, they are today widely rejected and no longer at the center of interest."*27

In Defense of the Kalam Cosmological Argument - William Lane Craig

*24 - 24 See Robert Weingard, “Do Virtual Particles Exist?’ in Proceedings of the Philosophy of Science Association, 2 vols., ed. Peter Asquith and Thomas Nichols (East Lansing, Mich.: Philosophy of Science association 1982), I: 235-242.
*25 25 Alexander W. Stern, “Space, Field, and Ether in Contemporary Physics,” Science 116 (1952): 493. Stern is even willing to speak of the quantum vacuum as a sort of ether.
*26 See discussion in William Lane Craig and Quentin Smith, Theism, Atheism, and Big Bang Cosmology (Oxford: Clarendon Press, 1993), pp. 125-129, 148-157.
*27 See William Lane Craig, “The Caused Beginning of the Universe: a Response to Quentin Smith,” British Journal for the Philosophy of Science 44 (1993): 632-633, 635.




"But as a counterexample to (1'), Smith's use of such vacuum fluctuations is highly misleading. For virtual particles do not literally come into existence spontaneously out of nothing. Rather the energy locked up in a vacuum fluctuates spontaneously in such a way as to convert into evanescent particles that return almost immediately to the vacuum. As John Barrow and Frank Tipler comment, ". . . the modern picture of the quantum vacuum differs radically from the classical and everyday meaning of a vacuum-- nothing. . . . The quantum vacuum (or vacuua, as there can exist many) states . . . are defined simply as local, or global, energy minima (V'(O)= O, V"(O)>O)" ([1986], p. 440). The microstructure of the quantum vacuum is a sea of continually forming and dissolving particles which borrow energy from the vacuum for their brief existence. A quantum vacuum is thus far from nothing, and vacuum fluctuations do not constitute an exception to the principle that whatever begins to exist has a cause. It therefore seems that Smith has failed to refute premiss (1')."


"Nor does the comparison of the universe's origin to the spontaneous production of a virtual particle serve to render these models plausibly realistic. For if this comparison is meant to be reasoning by analogy, then it seems extraordinarily weak, since the disanalogies between the universe and a virtual particle are patent. If we are to believe with Tryon [1973] that the universe literally is a virtual particle, then this seems even more preposterous, since the universe has neither the properties nor behavior of a virtual particle. One might ask, too, why quantum fluctuations are not now spawning universes in our midst? Why do vacuum fluctuations endure so fleetingly rather than grow into mini- universes inside ours?"

The Caused Beginning of the Universe: A Response to Quentin Smith - William Lane Craig

Adrian said...

Charlie,

An object could exist and be complex "in any way" yet lack proper physical parts. You have not identified any problems with this possibility.

Actually, you offered a system which required many independent dimensions, one dimension for each bit to be stored. The object in this instance is merely a placeholder, all the data is in the space and the space is extremely complex. You argue that the object is complex but this is incorrect - the space is complex while the object is simple and for this reason your example fails. I also listed a number of problems with this possibility including data representation, creating & manipulating the number of dimensions as you add data, and trying to retrieve the data from this space.


as his position now, confirmed by science, is that the presence of a quantum mechanical vacuum would be a sufficient reason for the existence of virtual particles -- virtual particles are probabilistically caused by the vaccua.

As I said earlier: "Talking about the presence (or more accurately the absence) of fields as "causing" VPs is even worse. There was no agent, no actor, no trigger, no impetus, nothing which we would describe as a cause. They arise due to the nature of quantum uncertainty in fields - this really stretches causality. If Christian apologists are willing to water down the subject of causality this far, they have reduced the "cause" of the Big Bang to this same level, a "cause" no more significant than the void between fields, not even an agent or an entity but a void or a line in a physical theory."


It could be that you wish to water down the term "cause" to abandon the idea of agents and interactions of any sort, certainly you would have to do so in order to defend Kalam. If you do so, then the "cause" becomes natural physical laws and so the only conclusion that could be drawn from Kalam is that the universe was "caused" by natural physical laws. Is that what you wish?


"The microstructure of the quantum vacuum is a sea of continually forming and dissolving particles which borrow energy from the vacuum for their brief existence. A quantum vacuum is thus far from nothing, and vacuum fluctuations do not constitute an exception to the principle that whatever begins to exist has a cause."

Craig has it backwards and he reaches a mistaken conclusion.

The quantum vacuum results in VPs precicely because nature prevents "nothing", that whenever we get close to a true nothing, matter springs into existence uncaused. The evidence demonstrates that matter truly comes out of nothing and Craig is just observing that after matter is creating we no longer have nothing. Well no kidding.


Defenders of Kalam find themselves in the position of making an assertion and having absolutely no evidence. Instead of supporting their claims, the best apologists are reduced to ward off attacks and their doing so by redefining their terms under the covers. What does cause mean to the English speaking world? "A person or thing that acts, happens, or exists in such a way that some specific thing happens as a result". What does cause mean to Craig and Kalam defenders? You tell me. I'm guessing it's something like "any physical laws or preconditions which existed before an event occurred."

As I've said many times already, VPs lack any agent, entity, interaction, connection, trigger, actor or event which leads to their creation. That's what I mean when I say they lack a cause. If you wish to contest this, please do us the courtesy of defining "cause" so we all know what P1 in Kalam really means.

Adrian said...

David,

Thanks for the elaboration.

First I agree with the distinction between necessary and sufficient conditions. Obviously a quantum vacuum is a physically necessary condition of a virtual particle's existence, therefore in that sense VP's have causes.

That's fine. Just so we're both very clear on this point, you are redefining "cause". To make this perfectly evidence, would you agree to change P1 in Kalam to read:

P1a - Everything which begins to exist has necessary preconditions.


That would clear up any confusion between your use of the word "cause" and the definitions found in any dictionary.

I would still protest of course, since it should read "Everything that we know of which began to exist has had necessary preconditions". Still, in order to see where you're going I'd grant P1a for the sake of argument.


I don't want to debate Craig since he's not here to defend himself, so I'll just mention one of his bigger mistakes:

The quantum vacuum is a sea of fluctuating energy which gives rise to virtual particles.

The quantum vacuum is a sea of fluctuating energy, but the fluctuating energy are the VPs, not what gives rise to them. The energy does not exist and VPs are created from it, no matter or energy exists in this region and VPs spring into existence and it is their energy which results in fluctuating energy. The reason VPs exist is because zero or nothing is a precise value for a field which is prohibited, so they wink into existence maintaining a minimum uncertainty.

David B. Ellis said...


Now Mr. Ellis agreed that the proposition was true, so I continued on to discuss the implications of the argument with him.


I was afraid you might misinterpret what I said---and you seem to have done just that. I didn't agree with the Kalam Cosm. argument (KCA from now on). I said you and I are in agreement that the big bang had a cause (probably, at least). There's a big difference there. KCA states:

P1 Everything that begins to exist has a cause
P2 The universe began to exist
C Therefore, the universe had a cause

First, I find P1 to be, in all likelihood, irrelevent. I doubt physical reality as a whole had a beginning.

P2 depends partly on the definition of universe. If you're referring to the big bang event and all that has proceeded from it then, yes, that event had a starting point. But I doubt very much that physical reality as a whole ever didn't exist. I think some event, maybe a quantum fluctuation, probably caused the big bang (and I wouldn't interpret this as an uncaused event, as Tyro seems to, only a chaotic one---but I guess that depends partly on what one means by caused---the diffence may be more one of semantics than substance).

Basically, my refutation of the KCA consists in pointing out that there are reasonable naturalistic alternatives to theistic creation to account for the big bang and so this argument does not provide any basis for thinking it any more likely that God exists.

To name just one example of a naturalistic alternative to God as first cause: the idea that our big bang was the eruption within the quantum foam---which is, as I understand it, a hypothesized "sea" of quantum fluctuations. If this model is correct there would likely be a vast number of universes---maybe an infinity of them. So this model would provide a naturalistic alternative to both first cause arguments for theism and the anthropic coincidences argument for theism.

Of course, there are quite a few other possibilities being debated by cosmologists. We don't yet know what model is correct. It might even be one that hasn't been thought up yet.

Its a wide open field and we (theists and naturalists alike) can only speculate at this point. And, unfortunately, one can not build a solid case for theism (or anything else) on mere speculation.

David Parker said...

David Ellis,

First, I find P1 to be, in all likelihood, irrelevent. I doubt physical reality as a whole had a beginning.

What leads you to doubt that physical reality had a beginning? Scientific data?
Do you think we're living in an actually infinite universe?

As to the relevence of P1, I am confused by your dismissal of it. It is rather germane to any discussion of cosmology.

P2 depends partly on the definition of universe. If you're referring to the big bang event and all that has proceeded from it then, yes, that event had a starting point. But I doubt very much that physical reality as a whole ever didn't exist. I think some event, maybe a quantum fluctuation, probably caused the big bang (and I wouldn't interpret this as an uncaused event, as Tyro seems to, only a chaotic one---but I guess that depends partly on what one means by caused---the diffence may be more one of semantics than substance).

I define the universe as everything that physically exists: the entirety of space and time, all forms of matter, energy and momentum, and the physical laws and constants that govern them.

Basically, my refutation of the KCA consists in pointing out that there are reasonable naturalistic alternatives to theistic creation to account for the big bang and so this argument does not provide any basis for thinking it any more likely that God exists.

So far you seem to have only asserted that physical reality as a whole is beginningless. The notion that a quantum flucation caused the big bang will need to be fleshed out, if you truly with to establish an alternative explanation to the theistic account.

To name just one example of a naturalistic alternative to God as first cause: the idea that our big bang was the eruption within the quantum foam---which is, as I understand it, a hypothesized "sea" of quantum fluctuations. If this model is correct there would likely be a vast number of universes---maybe an infinity of them. So this model would provide a naturalistic alternative to both first cause arguments for theism and the anthropic coincidences argument for theism.

a) a multi-verse is questionably uneccesary by Occam's razor
b) a multi-verse simply demands more explanation
c) the universe generation mechanism also needs an explanation
d) have you seen the arguments against an actual infinite's existence?

Your model just creates more questions and answers non of theism's. You haven't even identified a first cause, so how can you possibly hope to provide an alternative to the first cause argument of theism? Moving it back a step and making it more complicated only make your case more difficult to establish.


Of course, there are quite a few other possibilities being debated by cosmologists. We don't yet know what model is correct. It might even be one that hasn't been thought up yet.

Do you think any theory will ever concretely demonstrate that the universe always existed?

Its a wide open field and we (theists and naturalists alike) can only speculate at this point. And, unfortunately, one can not build a solid case for theism (or anything else) on mere speculation.

This is rhetoric. You have just admitted that naturalists speculate as well, so apparently you don't think either side has a case to be made?

David Parker said...

David Ellis,

I would also be interested in what you call an "anthropic coincidences argument for theism."

I am only familiar with atheist rendition that basically says, "well we are here to observe the universe so we shouldn't be suprised to find it."

David B. Ellis said...


What leads you to doubt that physical reality had a beginning?


I share the intuition (though I consider it no more than that) that reality didn't come into being from total nothingness.


As to the relevence of P1, I am confused by your dismissal of it. It is rather germane to any discussion of cosmology.


It seems more intuitively plausible that there is some eternal reality. Both of us share that intuition. So it comes down to a question of whether physical reality could be eternal (as in the sea of quantum fluctuations) as you believe your God to be.

Unless you can give good reason to think otherwise then P1 can be disregarded since neither of us are contending reality popped into existence from nothingness.

P1 isn't a point of disagreement between us so what would be the point of debating it?


The notion that a quantum flucation caused the big bang will need to be fleshed out, if you truly with to establish an alternative explanation to the theistic account.


Why should my hypothesis have to be more "fleshed out" than yours?

And "god did it" isn't exactly a detailed mathematically precise hypothesis. You seem to be requiring of the opposing side more than you demand of your own.



a) a multi-verse is questionably uneccesary by Occam's razor


A deity is not favored over a multiverse as an explanation for anthropic coincidence by occam's razor.

Think about it. The multiverse hypothesis explains the issue in question in terms of something we already know to exist---space-times emerging from a big bang. It just posits more than one.

YOUR hypothesis on the other hand posits a whole other order of being---supernatural entities. A deity, in this case. Something we have no reason to think exists.

Occam's razor clearly favors the multiverse over a deity (or deities).


d) have you seen the arguments against an actual infinite's existence?


Yes. I've read a lot of William Lane Craig's writings on this topic.

I was unimpressed.

Besides, my hypothesis doesn't require an infinite universe or multiverse so there isn't much reason to go into that anyway.


Your model just creates more questions and answers none of theism's.


You'll have to elaborate on that. As it stands its just an unsupported claim.


You haven't even identified a first cause, so how can you possibly hope to provide an alternative to the first cause argument of theism?


The quantum sea is the first cause in the naturalistic alternative I mentioned.


Do you think any theory will ever concretely demonstrate that the universe always existed?


Probably not. But I don't have to prove an alternative model for the KCA to fail to make a case for theism. I just have to show that naturalistic alternatives are as reasonable.


This is rhetoric. You have just admitted that naturalists speculate as well, so apparently you don't think either side has a case to be made?


The KCA claims that theism is the best (or in some versions, the only possible) explanation for what caused the big bang. I have explained that there are plausible naturalistic alternatives. This is all that must be done for the KCA to fail.


so apparently you don't think either side has a case to be made?


Don't attribute to my argument more than it contained. I did not say that all arguments for or against theism are just speculation.

I said that, concerning the cause of the big bang, all we can do is speculate.

These are two very different propositions.

David Parker said...

Regarding the notion that virtual particles violate causality:

http://math.ucr.edu/home/baez/physics/Quantum/virtual_particles.html

David Parker said...

David Ellis,

Unless you can give good reason to think otherwise then P1 can be disregarded since neither of us are contending reality popped into existence from nothingness...P1 isn't a point of disagreement between us so what would be the point of debating it?

As a theist I do contend that God created the physical universe ex nihilo (out of nothing). So when you use terms like "reality" I am a bit confused as to what you are referencing. P1 does not include God. Therefore, we do disagree on P1, and there is a point in debating it since you deny P1.

Why should my hypothesis have to be more "fleshed out" than yours?

Haha well first you stated you are attempting to offer an alternate explanation, so at minimum you need to establish what you're proposing is both a) an alternative and b)explanatory.

If you can't substantiate the claim, that is fine. I am just going by what you say.


And "god did it" isn't exactly a detailed mathematically precise hypothesis. You seem to be requiring of the opposing side more than you demand of your own.

You don't understand the KCA if you think "god did it" is what is being argued.

A deity is not favored over a multiverse as an explanation for anthropic coincidence by occam's razor.

Think about it. The multiverse hypothesis explains the issue in question in terms of something we already know to exist---space-times emerging from a big bang. It just posits more than one.



a) What "is favored" requires argument and not merely your opine.
b) Positing more than one just raises the bar for the atheist who has to explain the multiverse and also the mechanism by which these multiverses are generated. And the question still sits on the table for you when you're finished, was that caused and if so by what?


YOUR hypothesis on the other hand posits a whole other order of being---supernatural entities. A deity, in this case. Something we have no reason to think exists.

The KCA simply posits a cause. And of course your assertion about what "we have no reason to think exists" clearly begs the question.


Occam's razor clearly favors the multiverse over a deity (or deities).

I love how you just make assertions. Here let me try: the moon is clearly made of green cheese!


Yes. I've read a lot of William Lane Craig's writings on this topic.

FYI the debate over actual infinity is far outside of Christian apologetics; Dr. Craig merely makes use of the conclusions for his own work.


The quantum sea is the first cause in the naturalistic alternative I mentioned.

So the quantum sea is a) physical and b)uncaused ?


The KCA claims that theism is the best (or in some versions, the only possible) explanation for what caused the big bang. I have explained that there are plausible naturalistic alternatives. This is all that must be done for the KCA to fail.

A best explanation is not defeated by a merely plausible counter-example.

Adrian said...

Regarding the notion that virtual particles violate causality:

How do you think this page supports your claims?

I can see it mention "causality" but if you read you'll see that it is only discussing whether VPs violate locality constraints. It certainly doesn't agree with any of your claims that anything "causes" VPs to exist!

I have already agreed that VPs do not violate locality in a reply to Eric: "The question of "causality" grows difficult at quantum levels and so discussions about violations are often translated to questions about violating "locality". VPs do not violate locality."

David Parker said...

"This "superluminal" propagation had better not transmit any information if we are to retain the principle of causality. I'll give a plausibility argument that it doesn't in the context of a thought experiment."

Adrian said...

"This "superluminal" propagation had better not transmit any information if we are to retain the principle of causality. I'll give a plausibility argument that it doesn't in the context of a thought experiment."

What bearing do you imagine this has on your argument?

David Parker said...

Tyro,

The article has no relation to "my argument." Apparently you already agree with it's conclusion. Sorry, I have not been reading Eric's interaction with you.

Adrian said...

Apparently you already agree with it's conclusion.

Yes, I agree with it, why wouldn't I? It is irrelevant to this discussion.

David Parker said...

Tyro,

So your position is that a sufficient condition is required to explain VPs right? I don't suspect that physics is going to come up with that anytime soon, but in my opinion VPs are basically just a word for something physicists haven't figured out yet.

At any rate, since a necessary condition has been established for VPs why are you now demanding a sufficient one? Did not Dr. Craig himself point to a physically necessary condition?

So why are you requiring more of his argument than he did? Shouldn't you instead be showing that a sufficient condition is required for the KCA to work?

Adrian said...

David,

So your position is that a sufficient condition is required to explain VPs right? I don't suspect that physics is going to come up with that anytime soon, but in my opinion VPs are basically just a word for something physicists haven't figured out yet.

I don't understand your first sentence so I can't agree with it. As I've said, VPs were predicted long before they were detected and they're an integral part to one of the most successful theories in physics so I'd say they are figured out.

At any rate, since a necessary condition has been established for VPs why are you now demanding a sufficient one? Did not Dr. Craig himself point to a physically necessary condition?

Again, don't know what you're driving at here.

I agree that physical laws describe the creation of VPs and I've said that if you reword P1 to "Everything that we know had a beginning arose in accordance with physical laws" I would agree to it. It's precise and accurate and reduces the chance of any bait and switch with definitional changes.

How would you reword P1?

So why are you requiring more of his argument than he did? Shouldn't you instead be showing that a sufficient condition is required for the KCA to work?

So far we both agree that many things which began to exist did so in accordance with physical laws. For the sake of argument I'll agree that the universe arose in a similar fashion. Where does that leave us? The most the KCA can conclude is that the universe arose in accordance with natural laws.

Since there was no agent, actor or event which results in the appearance of a VP there is no need for one in the appearance of the universe. Hence no God.

Where do we disagree?

Adrian said...

Should say "no need for a God".

Russ said...

District Supt. Harvey Burnett,

You said, "Russ, You're a NUT." I truly am puzzled by what "NUT" could possibly mean in the context of your life, Harvey. Let's think about this a bit.

You call me a NUT, but it is you, not me, who sees in a newborn infant the dehumanizing doctrine, the curse, of original sin. Your assinine claim, about which you also claim you are absolutely certain, is that, through no fault of her own, this child - note that non-believers like me see a beautiful innocent child, not some vulgar vector of evil - has inherited blameworthiness, and is to be held responsible, for some terrible deed committed by a pair of ancient ancestors.

Fortunately, sane, moral people know that no such debt exists to be shouldered by our children. Those ancient ancestors exist only in myths, fables, fairy tales and legends, and no child, Harvey, not your children, not my children, not one single child anywhere bears responsiblity for those imagined transgressions from so long ago. Original sin survives today as a fabricated psychological pathology which pads the pockets of clergy as they dispense their version of an also fabricated cure. Sadly, the psychological wounds of many Christian parents are so deep that instead of protecting their little ones, they allow them to be similarly scarred, similarly maimed by morally inept prevaricators like you, District Supt. Pastor Harvey Burnett.

You call me a NUT, but it is you, not me, who would consider an auditory hallucination saying "To demonstrate your faith in me, Harvey, kill your children," to be a sign from what you imagine is your loving Christian god. I would bet if you did EXPERIENCE such an hallucination, you would not act on it. Why? Simple. You know full well that no one would believe you: not me, not other commenters on this blog, not the District Attorney, not the sure-to-be-mostly-Christian jury, and, depending on the state where you acted on your delusion if you chose to to do so, neither your fellow inmates nor the executioner would believe you. If you can be honest for a moment, Harvey, you would not believe it of anyone else either. So much for faith. The Bible and Abraham be damned, moral people, even your fellow self-proclaimed Bible-believing Christians, will accommodate Christianity's delusions only so far. You portray Christian faith, the kind that could justify murdering one's own child, as admirable, desirable, laudable, but transport Abraham into the modern era where essentially everyone, religious and non-religious alike has abandoned such ludicrous faith, and he is not a revered saint, he is recognized as what he truly is: a maniac.

You call me a NUT, but it is you, not me, who claims to be absolutely certain from your personal Christian EXPERIENCE that demons, ghosts, witches, angels, fallen angels, devils, and gods exist. Truly, Harvey, that does not speak well for your ability to put your personal EXPERIENCE into a real world context. While you state that you have recorded your experiences, I'm sure you would never allow those records to be exposed to less credulous examination. No different than other clergy backed into corners, you need a credulous congregation to respect the authority they give you in such matters. You're stuck, Harvey. You can never back down if those you have been goading into believing your eerie ghost stories are going to keep paying for your place of business. The people you minister to crave for causes other than themselves or reality to blame for their miseries, and, you, armed with an apothecary of evil spirits and their antidotes, dutifully provide.

Unfortunately, Harvey, when the frenetic mass hysteria of the exorcism is over, when the fevered shared frenzy of the faith healing has passed, and when the soothing sense of being center stage at a prayer meeting is gone, your patients are no less afflicted by reality than they were before. They may have less money to confront that reality, but they are not less afflicted by it. Whether you take a salary, or get your remuneration in other ways, matters not. What does matter is whether their current lives and future prospects are improved for having experienced your regime of spells and incantations. They should, but rarely do, see themselves that what you've done is pointless.

You call me a NUT, and from the ways you choose to misread and misinterpret the world, I can see exactly why you would call me that. I'm sure you see anyone not suckling the same pacifier as a NUT. Realize Harvey, if Christianity was true, then, as the clergy maintain, including those in my family, lives of Christians would stand out as being observably better than non-Christians. They're not. Christianity lives by delusion only.

Harvey, you also said, "..the ONLY thing I hate worse than an Ivory Tower religious person is an Ivory Tower atheist..." When I was a child, religion to me was mostly dumb, even though I played along. When I became an "Ivory Tower religious person," religion acquired a broader deeper fuller dumbness, and now that I've graduated to "Ivory Tower atheist," depending on the specific religion, sect, denomination, and individual congregation, religion ranges from innocuous to dumb and on through stupid, idiotic, corrupt, and immoral.

I rather like, Harvey, how you seized upon the word HATE to describe your views of both the "Ivory Tower religious person" and the "Ivory Tower atheist." One, the "Ivory Tower religious person," you hate, though he may be a Christian also, because he is more knowledgeable, better educated, better informed. The other, "Ivory Tower atheist," you hate because...oh, interesting...you hate because he is more knowledgeable, better educated, better informed. Realize that Mr. Loftus has been both your "Ivory Tower religious person" and your "Ivory Tower atheist."

I don't feel bad that you HATE me, Harvey, since I know that you hatefully characterize all of humanity, including yourself and your children, as being worthy of eternal damnation from conception on. Only after someone claims to accept your fables and the characters and bizzare occurrences therein - but only if they agree to see it precisely as you do, even other Christianities don't pass muster - do you think them worthy of more than that.

I on the other hand do not hate you, Harvey, but then I characterize all of humanity as being inherently worthy of all the best that life has to offer. I've seen the lengthy list from your church's homepage of those that you and your congregation HATE as part of how you define your congregation-specific version of Christianity. You start with a divisive doctrine of HATE, which you will say should be seen as love, yet you call other people nuts.

You stated, Harvey, "I put my MONEY, TIME, ENERGY and RESOURCES where my mouth is." Yes! Yes! Yes! Those things are real and even you recognize that they translate into real world good for others. Your efforts are no different than mine, Harvey. They pay off the same. You fully realize, though you are unable to admit it, that your version of a god is not doing these things, Harvey, you are. MONEY, TIME, ENERGY and RESOURCES are not free, they do not come from a god and as the economy worsens they can just as easily disappear. That you attribute your efforts to some god, simply confirms that your claims from the pulpit are simply superstitions. You know the god you share in delusion with your congregation is not the cause of what good might come from your affliation. It is the efforts by you and your congregation members that determines what difference there might be. You insist on upholding the group's delusion of a god, but you clearly see that your real efforts by real people alone are both the cause and the effect.

Harvey says, "...and in my SPARE TIME I come here to read and debate opinions of what I believe to be some good people (YOU OBVIOUSLY EXCLUDED)..." HATE, we get it, Harvey.

You said, "If this is proof of how an atheist thinks (And if I were an atheist I'd disavow your representation in any fashion)...then it is also proof of how they never give the benefit of the doubt but always questioning BEYONED reason..." Harvey, the benefit of the doubt should be granted only when there exists either an expressed or implied acknowledgement of doubt. You claim complete certainty, while insisting you be granted the benefit of the doubt. Your claim of certainty is flawed, and thus, unwarranted, as even many a Christian points out(no I haven't forgotten your HATE of the "Ivory Tower religious person"), but you will not admit to it. You claim to know with absolute certainty that your version of Christianity benefits its practitioners, a claim that is verifiably false, yet you maintain your claim of certainty. No benefit of the doubt is due you, Harvey.

Harvey, you stated, "What is it, you couldn't click the link on my blog regarding Health Insurance (Bottom link Left side) are you TRUTH challenged?" Any money you might make in hawking health insurance to members of your congregation and other Christians, is due to the science, polls, academic studies and mathematics, you so ardently disdain. The actuarial data is accrued, correlated, analyzed and converted into convenient, easy to navigate tables, almost exclusively by academic researchers. If it makes money for you, it's good science, good research, good studies, but if work is done that contradicts your mythologies, then you simply reject it while urging your already needy congregation to do the same. Clearly, when it puts money in your pocket, when it feeds your family, all that blab about assumptions and presuppositions go right out the window and you rely completely on those same techniques, processes, and methodologies that you claim should be rejected in the natural sciences. If that's not dishonorable, I don't know what is. The same tools of science that make it possible for you to reliably make money selling health insurance, are the same tools that validate the results of physics, chemistry, biochemistry, biology - including evolution, genetics, medicine and astronomy laboratory work, Pastor. It's another example of you intentionally misleading your congregation to keep them ignorant of real world science, the science that directly impacts their lives every single day. That's immoral.

Then, Reverend, you make statements that are simply farcical like, "Just unable to understand that Christians such as myself do more good in ONE MINUTE of their existence than people like you do in a lifetime???" In one minute, huh? That is impressive. It's notable, Harvey, that you are talking about Christians helping Christians, here in the US, but again, US Christians show no signs - validated exactly as are your actuarial tables - that they have any better life outcomes, than do non-Christians, or non-believers.

You said,

By the way the next time we sponsor a back to school program as I have also done for many years (I have the TV interview if you want proof) let me see if I can get you to match the $3,000 that I usually put in personally without taking a tax deduction for it...

Give me your email, since you have so much to say about charlatans and let's see who the REAL charlatan is on this board...Let's see if you can help people just for the sake of helping people with no promise of any return or personal benefit LIKE ME...

Your innate charlatan is showing again, Harvey. You say you give money. Why? Because your useless notion of a god does nothing. You do. You give money. The money is not free. You get the money selling health insurance to members of your congregation and other Christians. You give money while attributing it to your no-show god. If a program to which you donate is successful, it has you and other contributors to thank, not some deity. You then outright lie to people and claim that your version of a god made it a success.

Near the end of your comment you say,

As you can see, I don't have too much to say to you in way of arguments and I'd prefer that you not address me outside of the arguments and topics here because you prove that you're an idiot.


As long as you can suck that pacifier, Harvey, I know you'll feel perfectly comfortable. Banter about theology is easy; it's safe; it has no consequences. As I told you in "The Bizzaro Beliefs of Christianity," Christianity itself is so varied that the term "Christian" is virtually meaningless outside most individual congregations. You, Pastor, define what Christianity is to your members, and they, quite contentedly, no doubt, go blithely about the business of being "Harvey Burnett" Christians. They won't think too long or hard about other Christianities not being like them, or even that other Pentacostals are not like them.

But that's not all there is to it. While you embrace the means of science that afford you income, you tell your congregation to reject science, which is a grave disservice to us all. Studies done using those same methods reveal that your version of Christianity fares less well in most measures of social and personal health and well-being, yet, you reject them. The same tools that bring you money and expose those places where persons of your own sect and congregation are in want of assistance, are rejected by you because you don't like what they say. You love that the science makes you money, but you reject the same science's undeniable implication that your version of Christianity is not all you claim it to be. You don't like what the science suggests about you and your inept god.

You can't be honest and tell your congregation that their studying science would benefit them, as well as the rest of us, financially and intellectually. You can't tell them that through the study of science they might themselves solve the world's energy problems, revolutionize agriculture, or allow the once-paralyzed to walk again, none of which can be solved via superstition.

You act in charitable ways, then superstitiously claim that the benefits derive from your version of a god. In doing so you debase, degrade, and denigrate the expression of human compassion that is common to us all, and is observable everywhere there are people without respect to religious identity. You want to claim that Christian charity is somehow superior to that shown by non-Christians, but it is not. Humanitarian aid is the same regardless of the source.

You ended saying,

Sorry everyone, please ovelook this occurance but for those who know me on this board, they also know I've handled this jerk rather nicely as compared to others who've presented themselves in a similar manner...


Harvey, I wrote this because I care about your church members and how you adversely affect their lives. You can say you HATE me and call me a jerk all you like. I don't hate you, Harvey, but I do harbor great disdain for ignorance draped in clerical robes, bearing the weight of authority and wielding that authority in a way that reflects that it is out of touch with the needs of both those subjected to it directly as well as the greater human community.

District Supt. Harvey Burnett said...

Russ,

LIKE I SAID AND YOU'VE CONFIRMED YOU'RE A NUT, A COMPLETE IDIOT AND CRAZY!

PROOF IS IN THE PUDDING!


LATER.

District Supt. Harvey Burnett said...

Ooh, by the way Russ, do you care to tell us at what age you were when you were molested the first time? It will be a good case study for future discussions.

And don't give any of us this personal attack garbage after all you said about me...and BESIDES all that you know it's true that you were molested.

Now, we don't blame the victims. I'm sorry it happened to you and I pray even against what you think is your "free will" that it doesn't happen that you continue to perpetuate this immoral behavior on others.

Please tell the audience a little about your experience in this. I think it would go a long way in helping everyone to understand you a little better.

I'm certainly waiting.

Later.

Philip R Kreyche said...

Way to avoid every single point Russ just made, Harv.

a helmet said...

David B. Ellis,

do you love God?

David Parker said...

Since there was no agent, actor or event which results in the appearance of a VP there is no need for one in the appearance of the universe. Hence no God.

So do you propose that physical laws were around before the physical universe? Either matter is eternal or it isn't.

Adrian said...

David,

So do you propose that physical laws were around before the physical universe?

We have no evidence either way.

I guess that means we should change P1 to read:

P1a: To the best of our knowledge, all matter which began to exist did so in accordance with physical laws.

I wouldn't want anyone to take the leap and start talking about the origin of these physical laws when we have absolutely no understanding of their origin at all.


Either matter is eternal or it isn't.

It isn't. All of the matter in the universe arose during the Inflationary period after the BB.

David Parker said...

I find it very interesting that you believe physical laws came about prior to the physical materials themselves.

So you don't have a problem with invariant abstract entities?

Adrian said...

I find it very interesting that you believe physical laws came about prior to the physical materials themselves.

The laws governing the creation of matter guided the creation of matter, hence they must have been active prior to matter's creation. All of this talk about "being", "creating" and "governing" are metaphorical/anthropomorphic however.

So you don't have a problem with invariant abstract entities?

I don't know to what extent physical laws may vary and I'm reluctant to call them "entities" except metaphorically. I'm not contesting it, just saying that our understanding is virtually non-existent.

For the sake of the argument, I'll grant you whatever speculative ideas you wish unless it is contradicted by observation. Does that help you?

Adrian said...

David,

I've asked a few times. Just to make it clearer: how would you rephrase P1 to make it much clearer and more explicit?

Can you remove some of the ambiguity by eliminating "thing" (matter? energy? what is a "thing"?) and "cause"?

David Parker said...

Tyro,

Not sure how I would reword it. From my reading of Craig, it is a metaphysical (not purely physical) statement that essentially echoes "something doesn't come from nothing,"

Certainly a sea of quantum foam (or whatever you call it) doesn't constitute nothing.

Adrian said...

David,

From my reading of Craig, it is a metaphysical (not purely physical) statement that essentially echoes "something doesn't come from nothing,"

Sadly that's my take as well. It seems designed to avoid any empirical scrutiny or connection to the real world. It's a fine starting point for a course in logic but it's no use in helping us learn about what's real.


200 years ago I think this would have gone down well. Back then, "something" meant merely "matter", and "nothing" would be akin to the vacuum of space. Debates in this era were around where the matter in trees came from and where it went when the tree was burned. Did it come from "nothing"? No!

Today's world is vastly more subtle. "Things" expand to encompass neutrinos which can pass through the entire earth without reacting, quarks and photons. "Nothing" has become unthinkable as we learn that that whenever there is a void (such as between atoms) virtual particles appear to fill it.

KCA has become an antiquated curiosity, preserved only because the language is so vague and insubstantial that it resists initial attempts to attack it (only insofar as it resists attempts to understand it).


Certainly a sea of quantum foam (or whatever you call it) doesn't constitute nothing.

Well there's a problem. If you want to say that this doesn't count - and you're free to do so! - then you have absolutely no examples of anything coming from anything else and you have no examples of "nothing". You can't say that something never comes from nothing because we have absolutely no examples of nothing, indeed if Craig has his way, "nothing" would mean an absence of even the Universe. You're left trying to reason about events which aren't merely unobserved but inconceivable.



I thought that atheists were attacked for presuming to have absolute knowledge yet here is Craig giving us absolute statements about what definitely happens outside of our universe!

David Parker said...

you have absolutely no examples of anything coming from anything else and you have no examples of "nothing".

You seem to put a lot of stock in such a young scientific field.

I don't think we will be coming up with examples of "nothing" anytime soon. Duh.

Adrian said...

I don't think we will be coming up with examples of "nothing" anytime soon. Duh.

I think you misunderstand me. I actually agree with you! I was trying to say that, according to Craig, in order to find "nothing" we would have to make observations outside of our universe! I don't think we'll ever find examples to fit Craig's bizarre requirements.

So he's successfully prevented falsification but it also means his statement is meaningless, pulled out of nothing, totally unsupported or unsupportable. Just nonsense. (Which is why I'm more interested in discussing this with you than Craig - he's too far gone.)

David Parker said...

Tyro,

So you do believe something always existed then?

Adrian said...

Define "something" :)

The matter in our universe arose around 14 bya, but we don't have enough information to draw conclusions about the origin of the universe as a whole.

David Parker said...

Tyro,

Something - any thing. :-)

David Parker said...

What scientific explanation could there possibly be for the existence of physical laws? It seems the prevalent theory is that the physical laws don't exist and are merely descriptive with respect to how physical things behave.

But you definition must grant them some ontological status. You seem to rightly shy away from this position.